Google Groups no longer supports new Usenet posts or subscriptions. Historical content remains viewable.
Dismiss

I corpi cadono veramente tutti con la stessa accelerazione?

353 views
Skip to first unread message

Wakinian Tanka

unread,
Aug 16, 2019, 9:25:03 AM8/16/19
to
Due uguali manubri da ginnastica di massa m = 1,5 kg vengono fatti cadere simultaneamente da una altezza di 10 m, in aria.
Uno dei due pero' e' legato ad una lunga catena metallica (piu' di 10m) la cui altra estremita' e' legata ad una mensola posta alla stessa altezza da cui i manubri sono fatti cadere, e vicino ad esso (manubrio) come in figura:
https://i.postimg.cc/sDC5nv5R/IMG-20190815-172002.jpg
La lunghezza della catena e' piu' che sufficiente per permettere al secondo manubrio di toccare il suolo.

I due manubri toccano il suolo simultaneamente (per quanto si puo' apprezzare ad occhio nudo)?
Uno dei due tocca il suolo prima dell'altro?

Motivare la risposta.

--
Wakinian Tanka

Soviet_Mario

unread,
Aug 16, 2019, 10:24:02 AM8/16/19
to
Il 15/08/19 17:46, Wakinian Tanka ha scritto:
credo che non si possa dare nessuna risposta senza scomodare
eventuali attriti e resistenze della catena alla mobilità
angolare (visto che la caduta del peso comporta il
movimento, anche strisciante, di alcuni anelli che si
riposizionano).
Non ho la più tenue idea di quanto questi attriti possano
rallentare (perlomeno imho di accelerare non se ne parla
nemmeno in teoria), e mi sembra ben poco e forse
trascurabile. Ma boh ...

Anche se non c'entra assolutamente nulla mi hai però fatto
venire in mente che avevo letto su una pagina wiki che la
trasmissione cardanica (per motivi a me completamente
oscuri) ciuccia via per attriti una quota notevole della
coppia dell'albero attivo rispetto a quello trascinato. Mi è
venuto in mente perché ogni articolazione degli anelli di
catena è nella stessa configurazione spaziale di un singolo
snodo cardanico : due ellissi ortogonali articolati

però il contesto è diverso perché la c'è una resistenza
esterna su un albero ed un altro che esercita coppia, il che
significa attriti da contatto e pressioni tra i due anelli
notevoli
qui invece è tutto in caduta libera e può sottrarsi alle
pressioni come preferisce non ci sono gli stessi vincoli
geometrici attorno per cui non mi aspetterei le stesse
resistenze allo srotolamento ecco

>
> --
> Wakinian Tanka
>


--
1) Resistere, resistere, resistere.
2) Se tutti pagano le tasse, le tasse le pagano tutti
Soviet_Mario - (aka Gatto_Vizzato)

Giorgio Bibbiani

unread,
Aug 16, 2019, 10:48:02 AM8/16/19
to
Il 15/08/2019 17.46, Wakinian Tanka ha scritto:
> Due uguali manubri da ginnastica di massa m = 1,5 kg vengono fatti
> cadere simultaneamente da una altezza di 10 m, in aria. Uno dei due
> pero' e' legato ad una lunga catena metallica (piu' di 10m) la cui
> altra estremita' e' legata ad una mensola posta alla stessa altezza
> da cui i manubri sono fatti cadere, e vicino ad esso (manubrio) come
> in figura: https://i.postimg.cc/sDC5nv5R/IMG-20190815-172002.jpg La
> lunghezza della catena e' piu' che sufficiente per permettere al
> secondo manubrio di toccare il suolo.

Perché indichi la massa dei manubri (che comunque in prima
approssimazione non compare nella soluzione) e non anche
quella della catena?

> I due manubri toccano il suolo simultaneamente (per quanto si puo'
> apprezzare ad occhio nudo)? Uno dei due tocca il suolo prima
> dell'altro?

Provo a scrivere alcuni pensieri ma senza certezza...

Suppongo di poter trascurare le dimensioni delle maglie
della catena, scelgo un sistema di riferimento K
avente accelerazione g (scalare) verso il basso e tale
che tutti i corpi siano in quiete in K al tempo t = 0 s
in cui inizia la caduta, in K la mensola M e il
suolo hanno accelerazione g verso l'alto, quando
il suolo urta il manubrio di destra la catena, se
è abbastanza lunga, non è stata ancora del tutto
tirata dalla mensola, quindi l'urto dei manubri con
il suolo è simultaneo in questa prima approssimazione.
Seconda approssimazione: se i manubri sono tenuti fermi
per t < 0 s allora inizialmente la catena è in tensione
ed esercita una forza verso il basso sul manubrio
collegato, che urterà il suolo un po' prima dell'altro,
l'effetto non sarà macroscopico.
Terza approssimazione: durante il moto la catena
oscillerà a destra e a sinistra, se l'oscillazione
fosse abbastanza ampia potrebbe far sì che la catena
esercitasse una forza sul manubrio prima che questo
urtasse il suolo e l'effetto sarebbe ancora di
farlo urtare il suolo un po' prima dell'altro.

Ciao

--
Giorgio Bibbiani
(mail non letta)

Wakinian Tanka

unread,
Aug 16, 2019, 12:30:02 PM8/16/19
to
Il giorno venerdì 16 agosto 2019 16:24:02 UTC+2, Soviet_Mario ha scritto:
> ...
> credo che non si possa dare nessuna risposta senza scomodare
> eventuali attriti e resistenze della catena alla mobilità
> angolare (visto che la caduta del peso comporta il
> movimento, anche strisciante, di alcuni anelli che si
> riposizionano).
>
Lo avrei detto anch'io :-)
Ma in questo caso non e'... il caso ;-)
C'e'... qualcosa che va considerato (io non me lo sarei immaginato, benche' non sia qualcosa fuori della nostra portata).

--
Wakinian Tanka

Wakinian Tanka

unread,
Aug 16, 2019, 12:30:02 PM8/16/19
to
Il giorno venerdì 16 agosto 2019 16:48:02 UTC+2, Giorgio Bibbiani ha scritto:

> Perché indichi la massa dei manubri (che comunque in prima
> approssimazione non compare nella soluzione) e non anche
> quella della catena?
>


Hai ragione. La massa l'ho scritta solo per dare un'idea delle dimensioni: sono normali manubri da ginnastica (molto approssimativamente un 25 cm di lunghezza e 8 di diametro nelle parti eserne). La catena metallica e', tanto per avere un'idea, una normale catena tipo quella con cui si legherebbe un motorino (maglie di 3 cm circa). Ma il tipo, le dimensioni e la massa esatti non sono, naturalmente, essenziali.
>
> Provo a scrivere alcuni pensieri ma senza certezza...
>
> Suppongo di poter trascurare le dimensioni delle maglie
> della catena, scelgo un sistema di riferimento K
> avente accelerazione g (scalare) verso il basso e tale
> che tutti i corpi siano in quiete in K al tempo t = 0 s
> in cui inizia la caduta, in K la mensola M e il
> suolo hanno accelerazione g verso l'alto,
>
Anche la parte destra della catena, pero', cioe' quella che ciondola dalla mensola.
>
> quando il suolo urta il manubrio di destra la catena, se
> è abbastanza lunga, non è stata ancora del tutto
> tirata dalla mensola,

Questo... potrebbe non essere del tutto esatto ;-)


> Terza approssimazione: durante il moto la catena
> oscillerà a destra e a sinistra,
>
Si puo' trascurare l'oscillazione.
Ciao.

--
Wakinian Tanka

Wakinian Tanka

unread,
Aug 16, 2019, 1:30:03 PM8/16/19
to
Il giorno venerdì 16 agosto 2019 15:25:03 UTC+2, Wakinian Tanka ha scritto:
> Due uguali manubri da ginnastica di massa m = 1,5 kg vengono fatti cadere
> simultaneamente da una altezza di 10 m, in aria.
...
> La lunghezza della catena e' piu' che sufficiente per permettere al secondo
> manubrio di toccare il suolo.
>
Precisazione: la lunghezza della catena e' si sufficiente a far toccar terra al manubrio ma non troppo lunga da toccar terra prima di esso.

L'esperimento avviene in aria e quindi facilmente realizzabile in pratica, ma il risultato non dipende dalla presenza dell'aria (spieghero' dopo qualche risposta).

--
Wakinian Tanka

Soviet_Mario

unread,
Aug 16, 2019, 2:12:03 PM8/16/19
to
Il 16/08/19 17:50, Wakinian Tanka ha scritto:
non so, non l'ho menzionato perché mi parevano pippe
mentali, ma ho anche pensato ad "onde" più o meno
anarmoniche laterali che si generano nella catena che cmq
viene spinta anche lateralmente mentre si svolge, e che
possono come accennavo secondo me a Giorgio tirare
lateralmente e quindi accorciare la catena.
Di sicuro (facendo prove pratiche casuali) nei casi in cui
la catena è CORTA ti accorgi che il peso arriva sul fondo ma
la catena vibra lateralmente e fa dondolare per un po' la
massa. Una parte del moto può magari (secondo l'elasticità
del cavo) essere dovuta alla trazione, ma una parte è
sicuramente dovuta al modo laterale della catena tesa che
vibra lateralmente. Questo moto si deve pur generare in
qualche maniera quando una parte del filo spinge via l'altro
perché cmq prende delle accelerazioni anche non coassiali

Il contesto mi richiama alla memoria (ma non ricordo il
discorso) anche di annaffiare lontano per la massima
portata, nella parte di fenomeno che non dipende dall'angolo
sulla verticale, bensì nell'assetto dell'ugello (fermo,
oscillante, di quanto oscillante e con che frequenza) :
perché la coesione interna del getto prima di sfrangiare ha
delle influenze. La cosa è complicatissima. Lo sfrangiamento
riduce alquanto la gittata : i singoli spruzzi sono meno
penetranti della massa bulk. Poi c'è una cosa vantaggiosa a
flusso compatto dove la parte che esce spinge un po' la
colonna, ma con lunghezze di colonna non molto grandi già la
colonna (per attrito coll'aria) comincia a frenare la parte
appena espulsa e bisogna spostare di un po' facendo una
sorta di "purge" davanti all'ugello. Cmq il fenomeno è
troppo complicato per immaginare quante dannate variabili
influiscano con una canna reale o col getto
dell'idropulitrice. Normalmente ottimizzo "a caso" con
piccoli tentativi e vedo cosa succede.
Nell'idrodinamica si annida il Demonio. Basta solo guardare
una fiamma di candela e cercare di capire in che condizioni
si allunga e sta ferma e quando comincia a oscillare e con
che frequenza. E' frustrante quando cerchi di regolare degli
stoppini per leggere, perché quando oscilla la luminosità
sfarfalla mostruosamente. Lo fa anche in aria praticamente
ferma, basta il regime di flusso autogenerato. A volte si
stabilizza, per poco, poi subito crea turbolenza. Boh
Mi sembra la cosa come col getto "fisso" della canna, dove
una colonna di fluido troppo lunga rallenti al punto che
ogni nuova aliquota espulsa cerca di farsi largo e crea
turbolenza e il getto perde l'aspetto di zampillo filante e
perde gittata

Wakinian Tanka

unread,
Aug 16, 2019, 3:05:02 PM8/16/19
to
Il giorno venerdì 16 agosto 2019 19:30:03 UTC+2, Wakinian Tanka ha scritto:
> Precisazione...

Questa precisazione avrebbe dovuto arrivare subito dopo il mio primo post...
Spero di non aver confuso le cose.
Sia Soviet che Bibbiani comunque hanno parlato di qualcosa che ha a che vedere con la soluzione, ma per il momento voglio fare il vago :-)

--
Wakinian Tanka

JTS

unread,
Aug 16, 2019, 3:05:02 PM8/16/19
to
Noto una cosa: la catena, che inizialmente si muove, durante il movimento stesso si ferma. Questo vuol dire che la sua energia cinetica sparisce, e questo potrebbe influenzare il moto del manubrio. Non so ancora come fare il calcolo: per il momento tiro ad indovinare, quello libero tocca prima (la ragione e' che la catena puo' esercitare solo forze dirette verso l'alto).

Soviet_Mario

unread,
Aug 16, 2019, 6:00:02 PM8/16/19
to
Il 16/08/19 20:14, Wakinian Tanka ha scritto:
la mia curiosità è peraltro meno che blanda, perché so a
priori che non potrò mai capire una soluzione presentata con
tanto di equazioni :) :)

Franco

unread,
Aug 16, 2019, 7:18:02 PM8/16/19
to
On 8/15/2019 08:46, Wakinian Tanka wrote:

> I due manubri toccano il suolo simultaneamente (per quanto si puo' apprezzare ad occhio nudo)?
> Uno dei due tocca il suolo prima dell'altro?

Ha qualcosa a che vedere con il fenomeno mostrato qui?

https://www.youtube.com/watch?v=Q571Qr8l4uk


---
This email has been checked for viruses by AVG.
https://www.avg.com

Soviet_Mario

unread,
Aug 16, 2019, 8:54:03 PM8/16/19
to
Il 17/08/19 01:14, Franco ha scritto:
spettacolo !!!

però l'esempio non mia pare calzare interamente

lì nel video ci sono due tratti di catena (tesi) che
viaggiano in opposto senso di marcia e il tratto curvo che
li collega che crea una specie di "rotatore" attorno a un
asse fisso.

L'esempio di Wakinian mi pare più simile alla puleggia
mobile di una carrucola. Magari è uguale ma boh

Elio Fabri

unread,
Aug 17, 2019, 2:36:02 AM8/17/19
to
Giorgio Bibbiani ha scritto:
> Seconda approssimazione: se i manubri sono tenuti fermi per t < 0
> s allora inizialmente la catena è in tensione ed esercita una forza
> verso il basso sul manubrio collegato, che urterà il suolo un po'
> prima dell'altro,
Quindi secondo te se tengo in mano un mattone che ne tiene un altro
appeso con uno spago, quando lascio il mattone superiore questo inizia
a cadere con accel. > g?


--
Elio Fabri

Giorgio Bibbiani

unread,
Aug 17, 2019, 3:06:02 AM8/17/19
to
Idealmente e _teoricamente_ sì (l'effetto non
sarà macroscopico): se lo spago è in tensione e
si comporta come un corpo elastico, quando si
lascia il mattone superiore su questo agisce la
forza peso e in più, per una breve durata di
tempo, la forza esercitata dal corpo elastico che
inizialmente è ancora in tensione, il risultato
è che inizialmente l'accelerazione del mattone
superiore è maggiore di g (e quella del mattone
inferiore minore di g).

Pangloss

unread,
Aug 17, 2019, 3:20:02 AM8/17/19
to
[it.scienza.fisica 16 Aug 2019] Franco ha scritto:
> Ha qualcosa a che vedere con il fenomeno mostrato qui?
>
> https://www.youtube.com/watch?v=Q571Qr8l4uk

Spettacolare!
Ma la ripresa mi sembra poco onesta, avrei gradito uno "zoom out" che permettesse
di vedere il dislivello tra il bicchiere ed il pavimento sul quale si raccoglie
la catenella. Dalle didascalie mi pare di capire che tale dislivello sia 2.7 m
(ma che ragazza alta!) e che la velocita' media della catenella (lunga 49 m)
lasciata cadere con un peso in testa) sia grossomodo di circa 4 m/s (!).
Insomma, il filmato e' interessante, l'osservazione sulle onde anche, ma direi
che e' stato girato per stupire e che non c'entra con il problema di W.T.

--
Elio Proietti
Valgioie (TO)

Wakinian Tanka

unread,
Aug 17, 2019, 3:35:03 AM8/17/19
to
A questo punto posto il video con l'esperimento, in inglese:

https://youtu.be/X-QFAB0gEtE

--
Wakinian Tanka

Elio Fabri

unread,
Aug 17, 2019, 4:30:02 AM8/17/19
to
Giorgio Bibbiani ha scritto:
> Idealmente e _teoricamente_ sì (l'effetto non sarà macroscopico): se
> lo spago è in tensione e si comporta come un corpo elastico
In effetti hai ragione.
Addirittura l'accel. iniziale del mattone superiore è 2g, quella del
mattone inferiore è nulla.
Se invece di uno spago è una molla, di costante elastica k,
all'inizio, quando è tutto fermo, la molla sarà allungata di mg/k.
Appena lascio il mattone inizia un'oscillazione, di ampiezza mg/k e
periodo 2pi*sqrt(m/2k), che poi si smorza nel corso della caduta.

Per lo spago è diverso, perché quando arriva alla lunghezza di riposo
si allenta. Quindi dopo un quarto di periodo l'oscillazione cessa e
rimane la velocità relativa dei due mattoni raggiunta in quell'istante:
-g*sqrt(2m/k)
(se non ho perso qualche fattore 2).
Se la caduta dura abbastanza a lungo il mattone superiore raggiunge
quello inferiore.

Tutto ciò ha a che vedere col problema proposto da WT?
Ancora non l'ho capito...


--
Elio Fabri

Soviet_Mario

unread,
Aug 17, 2019, 7:30:03 AM8/17/19
to
Il 17/08/19 01:14, Franco ha scritto:
mi è venuta in mente con un po' meno confusione la
differenza più sostanziale tra i due esempi : nel video la
catena TIRA, nell'esempio di Cometa la catena spinge (non mi
riferisco alla forza totale netta che agisce sulle sue
sezioni, ma al differenziale di forza creato dal
raddrizzamento : un effetto simile a quando srotoli un
rotolo, ieri ne ho srotolato uno di lana di vetro)

Soviet_Mario

unread,
Aug 17, 2019, 7:42:02 AM8/17/19
to
Il 17/08/19 09:24, Wakinian Tanka ha scritto:
ammetto di non capire molto la spiegazione : cioè non
immagino come l'arrestarsi della catena (che passa da caduta
a stazionaria) si trasmetta come spinta in basso. Avrei
detto che si trasferisse come maggiore trazione sulla parte
superiore.

L'accenno alla frusta che fa il ragazzo ha a qualcosa a che
vedere con "la fisica della frustata" ?

effe

unread,
Aug 17, 2019, 7:55:01 AM8/17/19
to
Il 17/08/2019 13.29, Soviet_Mario ha scritto:

> mi è venuta in mente con un po' meno confusione la
> differenza più sostanziale tra i due esempi : nel video la
> catena TIRA, nell'esempio di Cometa la catena spinge (non mi
> riferisco alla forza totale netta che agisce sulle sue
> sezioni, ma al differenziale di forza creato dal
> raddrizzamento : un effetto simile a quando srotoli un
> rotolo, ieri ne ho srotolato uno di lana di vetro)

A me sembrano due fenomeni analoghi in fondo si tratta sempre di una catena.
Mi viene da pensare che nel caso del video c'è la forza del pavimento
diretta verso l'alto quando la catena tocca terra (infatti accelera
bruscamente da un certo punto in poi) e la forza di reazione diretta
perso il basso, lungo la catena, che si somma alla forza peso per cui la
catena accelera.
Nell'esempio di cui si sta discutendo, il soffitto esercita una forza
verso l'alto a cui reagisce una forza verso il basso, ancora lungo la
catena, che accelera oltre g il corpo legato. Ma non saprei proprio
dimostrare se è giusto :-)

Soviet_Mario

unread,
Aug 17, 2019, 8:18:03 AM8/17/19
to
Il 17/08/19 13:44, effe ha scritto:
> Il 17/08/2019 13.29, Soviet_Mario ha scritto:
>
>> mi è venuta in mente con un po' meno confusione la
>> differenza più sostanziale tra i due esempi : nel video la
>> catena TIRA, nell'esempio di Cometa la catena spinge (non mi
>> riferisco alla forza totale netta che agisce sulle sue
>> sezioni, ma al differenziale di forza creato dal
>> raddrizzamento : un effetto simile a quando srotoli un
>> rotolo, ieri ne ho srotolato uno di lana di vetro)
>
> A me sembrano due fenomeni analoghi in fondo si tratta sempre di una catena.

già : due fenomeni analoghi dal risultato opposto :\

> Mi viene da pensare che nel caso del video c'è la forza del pavimento
> diretta verso l'alto quando la catena tocca terra (infatti accelera
> bruscamente da un certo punto in poi) e la forza di reazione diretta
> perso il basso, lungo la catena, che si somma alla forza peso per cui la
> catena accelera.
> Nell'esempio di cui si sta discutendo, il soffitto esercita una forza
> verso l'alto a cui reagisce una forza verso il basso, ancora lungo la
> catena, che accelera oltre g il corpo legato. Ma non saprei proprio
> dimostrare se è giusto :-)
>


Wakinian Tanka

unread,
Aug 17, 2019, 8:35:02 AM8/17/19
to
Il giorno sabato 17 agosto 2019 13:42:02 UTC+2, Soviet_Mario ha scritto:
> Il 17/08/19 09:24, Wakinian Tanka ha scritto:
> > A questo punto posto il video con l'esperimento, in inglese:
> >
> > https://youtu.be/X-QFAB0gEtE
> >
> ammetto di non capire molto la spiegazione : cioè non
> immagino come l'arrestarsi della catena (che passa da caduta
> a stazionaria) si trasmetta come spinta in basso. Avrei
> detto che si trasferisse come maggiore trazione sulla parte
> superiore.
>
Il ragazzo, piu' o meno, dice (non ricordo le parole, sto improvvisando) :



<<la porzione di catena attaccata al manubrio e che sta sotto di esso, cade, ma poi il suo moto verso il basso viene arrestato dall'altra porzione di catena, quella connessa alla ringhiera. Cosicche' la porzione di catena che stava cadendo riceve dall'altra porzione una forza verso l'alto che quindi diventa una /tensione/nella catena, che si trasmette anche al manubrio, che quindi viene tirato giu' da una forza supplementare (oltre al peso del manubrio) >>.
>
> L'accenno alla frusta che fa il ragazzo ha a qualcosa a che
> vedere con "la fisica della frustata" ?
>
Direi proprio di si ;-)

--
Wakinian Tanka

JTS

unread,
Aug 17, 2019, 9:10:02 AM8/17/19
to
Ho visto il video fino al punto in cui il manubrio tocca terra, senza
ascoltare la spiegazione.

Ho fatto un calcolo utilizzando la

d(mv)/dt = mg

formula sulla quale non ho nessuna intuizione (e non mi sento neanche
sicuro che si applichi a questo caso), ma sono abbastanza curioso da
voler vedere dove portano i calcoli.

Indico con x la posizione del manubrio. La posizione iniziale e' x = 0 e
l'asse x e' diretto verso il basso.

Il corpo costituito dal

manubrio + "pezzo di catena compreso tra il manubrio e punto in cui la
catena gira"

cade con velocita'

v = dx/dt

e ha massa

m = m0 + (l/2 - x/2) * rho

dove m0 e' la massa del manubrio, l e' la lunghezza totale della catena,
rho e' la massa della catena per unita' di lunghezza. Per controllo,
nella posizione iniziale (x = 0) la massa e' m = m0 + l/2 * rho e in
quella finale (x = l) la massa e' m = m0.

La d(mv)/dt = mg e' quindi

- v^2 * rho/2 + m * dv/dt = m * g

(se ho fatto bene i conti)

cioe'

m * dv/dt = m * g + v^2 * rho/2

che e' anche

dv/dt = g + v^2 * rho/(2*m0 + (l-x)*rho)

Questa equazione va bene per rho = 0, per rho molto grande diventa

dv/dt = g + v^2 /(l-x)

la quale non mi convince.

Wakinian Tanka

unread,
Aug 17, 2019, 10:50:02 AM8/17/19
to
Il giorno sabato 17 agosto 2019 15:10:02 UTC+2, JTS ha scritto:
...
> Ho visto il video fino al punto in cui il manubrio tocca terra, senza
> ascoltare la spiegazione.
>
Questo si chiama voler capire la fisica! :-)
>
> Ho fatto un calcolo utilizzando la
> d(mv)/dt = mg
> formula sulla quale non ho nessuna intuizione (e non mi sento neanche
> sicuro che si applichi a questo caso),

... in quanto fai variare la massa.
Be', tutto dipende da come uno la applica, bisogna stare particolarmente attenti. Devo ammettere che non ho approfondito questa tua schematizzazione, pero' nel calcolo in fondo fai un'approssimazione non lecita, vedi qui sotto.

> dv/dt = g + v^2 * rho/(2*m0 + (l-x)*rho) (1)
>
> Questa equazione va bene per rho = 0, per rho molto grande diventa
> dv/dt = g + v^2 /(l-x)
>
No!
>
> la quale non mi convince.
>
Ci credo che non ti convince: quando x = l implicherebbe accelerazione infinita!
Ma l'approssimazione che fai non e' lecita proprio perche' nella (1) il termine 2*m0 /non puoi trascurarlo/ rispetto a (l-x)*rho) quando x e' vicino ad l.

--
Wakinian Tanka

JTS

unread,
Aug 17, 2019, 11:35:03 AM8/17/19
to
Am 17.08.2019 um 15:42 schrieb Wakinian Tanka:
> Il giorno sabato 17 agosto 2019 15:10:02 UTC+2, JTS ha scritto:

>>
>> Ho fatto un calcolo utilizzando la
>> d(mv)/dt = mg
>> formula sulla quale non ho nessuna intuizione (e non mi sento neanche
>> sicuro che si applichi a questo caso),
>
> ... in quanto fai variare la massa.
> Be', tutto dipende da come uno la applica, bisogna stare particolarmente attenti. Devo ammettere che non ho approfondito questa tua schematizzazione, pero' nel calcolo in fondo fai un'approssimazione non lecita, vedi qui sotto.

Vero, hai ragione.

>
>> dv/dt = g + v^2 * rho/(2*m0 + (l-x)*rho) (1)
>>
>> Questa equazione va bene per rho = 0, per rho molto grande diventa
>> dv/dt = g + v^2 /(l-x)
>>
> No!
>>
>> la quale non mi convince.
>>
> Ci credo che non ti convince: quando x = l implicherebbe accelerazione infinita!
> Ma l'approssimazione che fai non e' lecita proprio perche' nella (1) il termine 2*m0 /non puoi trascurarlo/ rispetto a (l-x)*rho) quando x e' vicino ad l.
>

Allora diventa per x vicino a l e rho qualunque

dv/dt = g + v^2 *rho /(2*m0)

e di nuovo pare che scegliendo rho grande l'accelerazione non sia
limitata. Forse e' possibile (il termine v^2 indica che stiamo bloccando
rapidamente parti di catena pesanti).

Carlo Pierini

unread,
Aug 17, 2019, 12:15:03 PM8/17/19
to
CARLO

Il manubrio con la catena toccherà il suolo in leggero ritardo rispetto all'altro, perché gli anelli di metà della catena devono ruotare di 180° e quindi freneranno la discesa del manubrio stesso.

ReBim

unread,
Aug 17, 2019, 3:05:02 PM8/17/19
to
Il giorno venerdì 16 agosto 2019 15:25:03 UTC+2, Wakinian Tanka ha scritto:
> Due uguali manubri da ginnastica di massa m = 1,5 kg vengono fatti cadere simultaneamente da una altezza di 10 m, in aria.

> Uno dei due pero' e' legato ad una lunga catena metallica (piu' di 10m) la cui altra estremita' e' legata ad una mensola posta alla stessa altezza da cui i manubri sono fatti cadere, e vicino ad esso (manubrio) come in figura:
> https://i.postimg.cc/sDC5nv5R/IMG-20190815-172002.jpg
> La lunghezza della catena e' piu' che sufficiente per permettere al secondo manubrio di toccare il suolo.
>
> I due manubri toccano il suolo simultaneamente (per quanto si puo' apprezzare ad occhio nudo)?
> Uno dei due tocca il suolo prima dell'altro?
>
> Motivare la risposta.
>
> --
> Wakinian Tanka


I baricentri dei 2 corpi ( manubrio e manubrio con catena ) non sono lasciati cadere dalla stessa altezza. lo sono i manubri, ma il baricentro del (manubrio con catena) è più basso.
Ovvio che non ci sia simultaneità nel contatto con il terreno-

ReBim

Maurizio Frigeni

unread,
Aug 17, 2019, 4:15:02 PM8/17/19
to
JTS <pire...@hotmail.com> wrote:

> dv/dt = g + v^2 * rho/(2*m0 + (l-x)*rho)

Ci sei andato vicino, ecco una trattazione per m0=0 (basata su una
diversa idea):

https://physics.stackexchange.com/questions/176686/can-someone-explain-this-solution-for-the-motion-of-a-falling-chain

M.

--
Per rispondermi via e-mail togli l'ovvio.

Soviet_Mario

unread,
Aug 17, 2019, 6:06:02 PM8/17/19
to
Il 17/08/19 20:59, ReBim ha scritto:
non capisco per niente l'obiezione
manubrio e catena non formano mica un corpo rigido ...

>
> ReBim

Pangloss

unread,
Aug 18, 2019, 3:50:02 AM8/18/19
to
[it.scienza.fisica 17 Aug 2019] Maurizio Frigeni ha scritto:
> Ci sei andato vicino, ecco una trattazione per m0=0 (basata su una
> diversa idea):
>
> https://physics.stackexchange.com/questions/176686/can-someone-explain-this-solution-for-the-motion-of-a-falling-chain

OK ma per capire qualitativamente il fenomeno non occorrono calcoli. :-)

Si consideri la configurazione della catena in un istante qualsiasi della caduta.
Gli anelli della parte bassa (curvilinea) subiscono una rapida diminuzione della
componente verticale della loro quantita' di moto. Cio' richiede che ai due
estremi di tale tratto curvo siano applicate opportune forze verso l'alto.
Per il principio di azione e reazione ne consegue che al fondo dei due tratti
rettilinei di catena agiscano analoghe forze verso il basso, che esercitano una
trazione sui rami verticali della catena.
Un lato e' vincolato, l'altro (quello con il manubrio) subisce un'accelerazione
aggiuntiva a quella di gravita'.

Elio Fabri

unread,
Aug 18, 2019, 4:24:03 AM8/18/19
to
Maurizio Frigeni ha scritto:
> Ci sei andato vicino, ecco una trattazione per m0=0 (basata su una
> diversa idea):
>
> https://physics.stackexchange.com/questions/176686/can-someone-explain-this-solution-for-the-motion-of-a-falling-chain
Secondo me quella "diversa idea" (conserv. dell'energia) è sbagliata.
Non credo affatto che nel moto della catena l'energia si conservi.

Pensate a che cosa fanno gli anelli: uno per uno l'anello in fondo
sotto il manubrio urta quello in fondo già fermo, ruota e si ferma a
sua volta.
A mio parere quell'urto è sostanzialmente anelastico, tranne forse
alla fine per la catena senza manubrio, dove la rotazione può essere
importante a causare un'inversione del moto.
Ma quando c'è il manubrio questo non accade e credo che il modello con
urti anelastici sia una miglore approssimazione.

Pangloss ha scritto:
> OK ma per capire qualitativamente il fenomeno non occorrono calcoli.
> :-)
Però occorre un qualche modello, e il tuo non mi pare rappresenti
correttamente la catena.

> Si consideri la configurazione della catena in un istante qualsiasi
> della caduta.
> Gli anelli della parte bassa (curvilinea) subiscono una rapida
> diminuzione della componente verticale della loro quantita' di moto.
Secondo me in una catena non c'è nessuna parte curvilinea.
Se ho ragione, questo distrugge il tuo ragionamento.

Ma continuiamo a tenerlo buono, per amore di discorso.
> ...
> Cio' richiede che ai due estremi di tale tratto curvo siano applicate
> opportune forze verso l'alto.
Ciò richiede solo che la *risultante* delle due forze sia verso l'alto.
Non ti dice niente sulle due forze separatamente.

Considera però che il tratto curvo dovrà anche ruotare, il che
significa che le due forze hanno momento non nullo rispetto a un punto
medio.
Questo già dice che se sono entrambe verso l'alto non possono essere
uguali, ma ancora non possiamo dire niente sulla loro intensità.

Credo proprio che gli argomenti qualitativi non portino a nulla.
Ci vuole un modello preciso, possibilmente vicino alla realtà, e al
tempo stesso calcolabile.


--
Elio Fabri

Maurizio Frigeni

unread,
Aug 18, 2019, 4:55:03 AM8/18/19
to
Elio Fabri <elio....@fastwebnet.it> wrote:

> Secondo me quella "diversa idea" (conserv. dell'energia) è sbagliata.
> Non credo affatto che nel moto della catena l'energia si conservi.

Anch'io lo credevo, però in quel libro si cita questo articolo del 1989
che dice il contrario:

https://aapt.scitation.org/doi/10.1119/1.16114

A dire il vero ho trovato anche quest'altro lavoro, che pare ripensare
l'approccio:

https://arxiv.org/pdf/1812.06942.pdf

JTS

unread,
Aug 18, 2019, 6:00:03 AM8/18/19
to
Am 18.08.2019 um 10:20 schrieb Elio Fabri:
> Maurizio Frigeni ha scritto:
>> Ci sei andato vicino, ecco una trattazione per m0=0 (basata su una
>> diversa idea):
>>
>> https://physics.stackexchange.com/questions/176686/can-someone-explain-this-solution-for-the-motion-of-a-falling-chain
>>
> Secondo me quella "diversa idea" (conserv. dell'energia) è sbagliata.
> Non credo affatto che nel moto della catena l'energia si conservi.
>
> Pensate a che cosa fanno gli anelli: uno per uno l'anello in fondo
> sotto il manubrio urta quello in fondo già fermo, ruota e si ferma a
> sua volta.
> A mio parere quell'urto è sostanzialmente anelastico, tranne forse
> alla fine per la catena senza manubrio, dove la rotazione può essere
> importante a causare un'inversione del moto.
> Ma quando c'è il manubrio questo non accade e credo che il modello con
> urti anelastici sia una miglore approssimazione.

In maniera analoga nel problema delle gocce nella nube gli urti sono
anelastici.

>
> Credo proprio che gli argomenti qualitativi non portino a nulla.
> Ci vuole un modello preciso, possibilmente vicino alla realtà, e al
> tempo stesso calcolabile.
>

Insisterei con la d(mv)/dt = F *se* capissi cosa vuol dire questa
equazione quando m e' variabile.

Carlo Pierini

unread,
Aug 18, 2019, 7:35:02 AM8/18/19
to
Il giorno sabato 17 agosto 2019 18:15:03 UTC+2, Carlo Pierini ha scritto:


> CARLO
>

> Il manubrio con la catena toccherà il suolo in leggero ritardo rispetto all'altro, perché gli anelli di metà della catena devono ruotare di 180° e quindi freneranno la discesa del manubrio stesso.

P.S.


Oltre al fatto che la (quasi) metà degli anelli deve ruotare di 180°, c'è da considerare che la catena, cadendo, tenderà a posizionarsi in linea verticale e quindi trascinerà verso destra anche il manubrio il quale, pertanto, allungando il percorso di caduta, toccherà il suolo in ritardo rispetto all'altro.

Pangloss

unread,
Aug 18, 2019, 7:35:02 AM8/18/19
to
[it.scienza.fisica 18 Aug 2019] Elio Fabri ha scritto:
> Pangloss ha scritto:
>> OK ma per capire qualitativamente il fenomeno non occorrono calcoli. :-)
> Però occorre un qualche modello, e il tuo non mi pare rappresenti
> correttamente la catena.
>> Si consideri la configurazione della catena in un istante qualsiasi della caduta.
>> Gli anelli della parte bassa (curvilinea) subiscono una rapida
>> diminuzione della componente verticale della loro quantita' di moto.
> Secondo me in una catena non c'è nessuna parte curvilinea.
> Se ho ragione, questo distrugge il tuo ragionamento.
> Ma continuiamo a tenerlo buono, per amore di discorso.
>> ...
>> Cio' richiede che ai due estremi di tale tratto curvo siano applicate
>> opportune forze verso l'alto.
> Ciò richiede solo che la *risultante* delle due forze sia verso l'alto.
> Non ti dice niente sulle due forze separatamente.
> Considera però che il tratto curvo dovrà anche ruotare, il che
> significa che le due forze hanno momento non nullo rispetto a un punto
> medio.
> Questo già dice che se sono entrambe verso l'alto non possono essere
> uguali, ma ancora non possiamo dire niente sulla loro intensità.

La mia risposta _qualitativa_ era assai laconica, voleva focalizzare la chiave
risolutiva del problema senza entrare nei dettagli.
Mi sembra chiaro che ho schematicamente considerato la catena come composta da
tre parti; due verticali ed in basso una parte curva di raccordo.
Non ho _mai detto_ che le due forze dirette verso l'alto siano uguali, ne' ho
fatto considerazioni sulla loro intensita', dipendente da molti dati strutturali
non dichiarati dall'OP o dal video.
Non ritengo di avere commesso errori, ma solo consapevoli omissioni.

Giorgio Pastore

unread,
Aug 18, 2019, 8:10:02 AM8/18/19
to
Il 18/08/19 13:04, Carlo Pierini ha scritto:
> Il giorno sabato 17 agosto 2019 18:15:03 UTC+2, Carlo Pierini ha scritto:
...
>> Il manubrio con la catena toccherà il suolo in leggero ritardo rispetto all'altro, perché ....

Se dai un'occhiata al video indicato da WT scoprirai che nell'
esperimento e' il manubio attaccato alla catena che tocca primo il suolo.

Giorgio Pastore

unread,
Aug 18, 2019, 8:55:02 AM8/18/19
to
Il 18/08/19 10:37, JTS ha scritto:
....
> Insisterei con la d(mv)/dt = F *se* capissi cosa vuol dire questa
> equazione quando m e' variabile.

Credo che la tua intuizion sia corretta: al di là dlle complicazioni
introdotte da catene, anelli, effetti elastici o anelastici, direi che
al cuore del problema c'e' la massa variabil del corpo che cade
collegato alla catena. Che rende il tutto analogo come schematizzazione
al problema della corda che pende da un estremo su un tavolo senza attrito.

In prima approssimazione possiamo considerarel' insieme manubrio+parte
della catena "che risale" come un corpo di massa variabile
M(t) = m + (m_c/2)*(1+z(t)/L)

dove m: massa manubrio
m_c: massa totale catena
L : lunghezza totale catena
z(t): quota al tempo t del manubrio
Possiamo pensare di attribuie quota z=0 al tempo iniziale (quota di
partenza) e quindi a fine caduta il manubrio sarà a quota -L.

l'equazione del moto di queto corpo a massa variabile sarebbe
d(M(t) z'(t))/dt = - M(t)*g

con g: accel. di gravità.

derivando e lasciando a sinistra dell' uguale il termine di
accelerazione z" si ottiene:

z"(t) = -g - (m_c/(2*L))*(z'(t))^2 / (m + (m_c/2)*(1+z/L))

che mostra immediatamente come, per tutto il tratto di caduta ( -L<z<0 )
l'accelerazione del corpo composito (manubio + tratto di catena) e' in
modulo maggiore di g.

Non ho provato a integrare analiticamente l'equazione, ma una soluzione
numerica mi dà un comportamento ragionevole per m_c ~ m e lunghezza di
caduta= lunghezza catena di qualche metro.

Il modello immediatamente successivo in "realismo", come già ossevato a
WT, dovrebbe essere quello di una catena con masse concentrate in punti
e snodi tra anelli rigidi (in pratica un pendolo composto).

Ma per simulare quella situazione avrei bisogno di un paio di ore che
non ho.

Se qualcuno ha mai giocato con algodoo (http://www.algodoo.com/), quel
software ha tutto quel che serve per simulare numericamente ed
accuratamente una catena e variare i parametri. Io pero' non sono mai
riuscito a usarlo con la stessa disinvoltura dei miei figli :-(

Elio Fabri

unread,
Aug 18, 2019, 9:12:03 AM8/18/19
to
Giorgio Pastore ha scritto:
> Credo che la tua intuizion sia corretta: al di là dlle complicazioni
> introdotte da catene, anelli, effetti elastici o anelastici, direi che
> al cuore del problema c'e' la massa variabil del corpo che cade
> collegato alla catena. Che rende il tutto analogo come
> schematizzazione al problema della corda che pende da un estremo su un
> tavolo senza attrito.
>
> In prima approssimazione possiamo considerarel' insieme manubrio+parte
> della catena "che risale" come un corpo di massa variabile
> M(t) = m + (m_c/2)*(1+z(t)/L)
>
> dove m: massa manubrio
> m_c: massa totale catena
> L : lunghezza totale catena
> z(t): quota al tempo t del manubrio
> Possiamo pensare di attribuie quota z=0 al tempo iniziale (quota di
> partenza) e quindi a fine caduta il manubrio sarà a quota -L.
>
> l'equazione del moto di queto corpo a massa variabile sarebbe
> d(M(t) z'(t))/dt = - M(t)*g
>
> con g: accel. di gravità.
Non ho capito niente.
Né la situazione fisica, né l'equazione che hai scritto.

Ipotesi:
1) sono rimbambito
2) mi occupo di troppe cose insieme
3) fa caldo
4) non sei un capolavoro di chiarezza.


--
Elio Fabri

Carlo Pierini

unread,
Aug 18, 2019, 11:00:03 AM8/18/19
to
Il giorno domenica 18 agosto 2019 14:10:02 UTC+2, Giorgio Pastore ha scritto:
> Il 18/08/19 13:04, Carlo Pierini ha scritto:
> > Il giorno sabato 17 agosto 2019 18:15:03 UTC+2, Carlo Pierini ha scritto:
> ...
> >> Il manubrio con la catena toccherà il suolo in leggero ritardo rispetto all'altro, perché ....

> GIORGIO
> Se dai un'occhiata al video indicato da WT scoprirai che nell'
> esperimento e' il manubrio attaccato alla catena che tocca primo il suolo.

CARLO

E' vero. L'avevo sottovalutato. La catena produce una sorta di "effetto frusta" che accelera il manubrio, ...ma che non sono in grado di esprimere matematicamente.

ReBim

unread,
Aug 18, 2019, 11:00:03 AM8/18/19
to
Penso (correggetemi se sbaglio) che si possa ragionare così.

Il sistema Manubio + catena (parte di catena tra il manubrio e la curva) ha, prima del moto, un baricentro situato tra il manubrio ed il punto di curva della catena.

Man mano che il manubrio cade, il baricentro di sposta verso il manubrio, in quanto si accorcia il tratto di catena che contribuisce alla massa totale del sistema.

Il baricentro si muove con accelerazione g, Ma la velocità del manubrio è la somma algebrica della velocità del baricentro + la velocità del baricentro rispetto al manubrio.
Per questo il manubrio agganciato tocca il suolo per prima.

Giorgio Pastore

unread,
Aug 18, 2019, 11:00:03 AM8/18/19
to
Il 18/08/19 15:07, Elio Fabri ha scritto:
....
> Ipotesi:
> 1) sono rimbambito
> 2) mi occupo di troppe cose insieme
> 3) fa caldo
> 4) non sei un capolavoro di chiarezza.
>

3+4+poca voglia da parte mia di fare l'"altro da-fare che incombe" ma il
tempo per divagare è poco. Potro' cercare di chiairire solo stasera.

JTS

unread,
Aug 19, 2019, 8:25:02 PM8/19/19
to
Am 18.08.2019 um 14:44 schrieb Giorgio Pastore:
> Il 18/08/19 10:37, JTS ha scritto:
> ....
>> Insisterei con la d(mv)/dt = F *se* capissi cosa vuol dire questa
>> equazione quando m e' variabile.
>
> Credo che la tua intuizion sia corretta: al di là dlle complicazioni
> introdotte da catene, anelli, effetti elastici o anelastici, direi che
> al cuore del problema c'e' la massa variabil del corpo che cade
> collegato alla catena. Che rende il tutto analogo come schematizzazione
> al problema della corda che pende da un estremo su un tavolo senza attrito.
>
> In prima approssimazione possiamo considerarel' insieme manubrio+parte
> della catena "che risale" come un corpo di massa variabile
> M(t) = m + (m_c/2)*(1+z(t)/L)
>
> dove m: massa manubrio
> m_c: massa totale catena
> L  : lunghezza totale catena
> z(t): quota al tempo t del manubrio
> Possiamo pensare di attribuie quota z=0 al tempo iniziale (quota di
> partenza) e quindi a fine caduta il manubrio sarà a quota -L.
>
> l'equazione del moto di queto corpo a massa variabile sarebbe
> d(M(t) z'(t))/dt = - M(t)*g
>


Ho capito finalmente la d(mv)/dt = F (dopo aver letto la pagina di
Wikipedia):

e' valida se l'unica interazione e' fra il corpo e la "massa eiettata" e
se la "massa eiettata" ha velocita' nulla dopo essersi staccata dal corpo.

Nel caso del manubrio e della catena e' vera la seconda ipotesi, ma la
prima no, perche' c'e' la parte di catena che si allunga.

Se si imposta l'equazione includendo la parte di catena che si allunga,
dovrebbe sempre rimanere un'incognita libera, perche' c'e' la forza del
vincolo.


Carlo Pierini

unread,
Aug 20, 2019, 2:05:03 AM8/20/19
to
Il giorno venerdì 16 agosto 2019 15:25:03 UTC+2, Wakinian Tanka ha scritto:
> Due uguali manubri da ginnastica di massa m = 1,5 kg vengono fatti cadere simultaneamente da una altezza di 10 m, in aria.

> Uno dei due pero' e' legato ad una lunga catena metallica (piu' di 10m) la cui altra estremita' e' legata ad una mensola posta alla stessa altezza da cui i manubri sono fatti cadere, e vicino ad esso (manubrio) come in figura:
> https://i.postimg.cc/sDC5nv5R/IMG-20190815-172002.jpg
> La lunghezza della catena e' piu' che sufficiente per permettere al secondo manubrio di toccare il suolo.
>
> I due manubri toccano il suolo simultaneamente (per quanto si puo' apprezzare ad occhio nudo)?
> Uno dei due tocca il suolo prima dell'altro?
>
> Motivare la risposta.

CARLO

La domanda cruciale è: chi fornisce al manubrio “incatenato” il sovrappiù di energia cinetica che gli permette di raggiungere il suolo per primo?


Questa energia gli sarà fornita dalla metà degli anelli della catena (lato manubrio) che dapprima scendono in caduta libera e poi, nel momento in cui saranno arrestati dagli anelli che li precedono (vincolati alla mensola) trasferiranno l’energia cinetica acquisita agli anelli successivi attraverso il processo che ho descritto sommariamente nel thread "Sbarra che cade...", e che riporto qui per comodità:
---------------------------



Immaginiamo che non si tratti di una sbarra, ma di un anello lungo e stretto, e che esso si muova - in direzione perpendicolare alla sua lunghezza - di moto rettilineo uniforme. Dobbiamo dimostrare che, nel momento in cui l’estremità A si arresta e si impernia in un vincolo immobile, l’estremità B passerà dalla velocità v ad una velocità più elevata. E sarà questo incremento di velocità il responsabile della “vittoria” del manubrio “incatenato” sul manubrio libero!




Intuitivamente si capisce che, se fino all’arresto dell’estremo A, la quantità di moto dell’anello era p=mv, l’arresto di A dovrà comportare necessariamente un aumento di velocità di B, visto che m e p devono restare costanti; in altre parole, la velocità perduta dal lato A deve essere acquistata dal lato B, se vogliamo che la velocità media sia ancora v e che il prodotto mv ci dia la stessa quantità di moto p (considerando nulla ogni dissipazione di energia). Ciò significa allora che, se A è passato da v a zero (cioè, da v a v-v), B dovrà passare da v a 2v (cioè da v a v+v). La -v in A deve diventare +v in B.




Ora, riportiamo questo ragionamento al caso del manubrio e della catena: questo incremento di velocità dell’estremità libera (lato-manubrio) di ogni anello si ripeterà in sequenza per ciascuno degli anelli a mano a mano che il manubrio stesso scende e che la catena si distende progressivamente in posizione verticale; incrementi di velocità che si sommeranno (in un effetto frusta) e che trascineranno il manubrio in una accelerazione via via crescente fino al contatto col suolo.

--------------

Giorgio Pastore

unread,
Aug 20, 2019, 2:05:03 AM8/20/19
to
Il 20/08/19 01:06, JTS ha scritto:
....
> Ho capito finalmente la d(mv)/dt = F (dopo aver letto la pagina di
> Wikipedia):
>
> e' valida se l'unica interazione e' fra il corpo e la "massa eiettata" e
> se la "massa eiettata" ha velocita' nulla dopo essersi staccata dal corpo.
>
> Nel caso del manubrio e della catena e' vera la seconda ipotesi, ma la
> prima no, perche' c'e' la parte di catena che si allunga.
>
> Se si imposta l'equazione includendo la parte di catena che si allunga,
> dovrebbe sempre rimanere un'incognita libera, perche' c'e' la forza del
> vincolo.

Le masse variabili sono un vero mal di testa visto che fanno parte dei
"cadaveri nell' armadio" non sempre discussi decentemente nei testi
didattici (anche universitari). Col risultato che spesso resta l'idea
che "basta" scrivere l'equazione del moto "alla Newton" come dp/dt = F
con p = m(t)*v(t). Come noti, condizione essnziale e' che la massa
"persa" abbia velocità nulla.

La mia è stata una scarnificazione del poblema all' elemento di base. E
ho trascurato anche quello che tu chiami forza del vincolo e che di
fatto è la tensione della catena. Ma se ti concentri sulla parte mobile,
credo che si possa giustificare di trascurarla in prima battuta (la
parte sotto tensione e' quella fissa, non quella in caduta; sulla
giunzione succederanno cose complicate ma sono convinto che in
approssimazione zero siano tutti dettagli che influiscono poco sul moto
della parte mobile).

Quando avrò tempo (ma non oggi) ricontrollerò sia i ragionamenti ( in
questo caso l'intuito ha preso il sopravvento ) sia i conti . La
dipendenza da (z')^2 mi sembra invece confermata da una veloce analisi
di quello (molto) che c'e' in letteratura, oltre, qualitativamente, dal
confronto con il video. BTW, la soluzione numerica dell' equazione mi
dà, in accordo col video, e coerentemente col termine (z')^2 che solo
nella seconda metà della caduta l'effetto di extra-accelerazione diventa
evidente nel confronto tra le traiettorie.

Giorgio

JTS

unread,
Aug 20, 2019, 10:20:03 AM8/20/19
to
On Tuesday, August 20, 2019 at 8:05:03 AM UTC+2, Giorgio Pastore wrote:

>
> La mia è stata una scarnificazione del poblema all' elemento di base.

Ho il sospetto che una soluzione generale non esista, e che il moto dipenda dai dettagli di come si piega la catena. Ma non sono sicuro

Per spiegarmi faccio un esempio che ha in comune solo il fatto che la soluzione dipende dai dettagli: quello della piastra che cade sulla pallina.



Il problema e' il seguente: c'e' una piastra pesante che cade verticalmente su una pallina elastica molto piu' leggera poggiata per terra. Supponiamo che nell'urto non venga dissipata energia (la quale pero' potrebbe essere dissipata dopo): a che altezza salta la pallina?



La soluzione si trova ammettendo che finche' piastra e pallina sono a contatto la velocita' di ogni punto della pallina sia lineare con la distanza dal suolo. In questo modo si trova che la velocita' del centro di massa della pallina e' uguale a meta' della velocita' della piastra e che la pallina salta ad un quarto dell'altezza da cui e' caduta la piastra.


L'ipotesi che la velocita' della piastra quando la pallina ha di nuovo la forma iniziale (dopo la compressione ed espansione) sia uguale alla velocita' al primo contatto la possiamo considerare "generale", ma l'ipotesi che la velocita' sia una funzione lineare della posizione no.



Lo stesso concetto potrebbe essere valido per il presente problema della catena: per trovare una soluzione potrebbe essere necessario dire come si comporta il pezzo di catena che si sta "svolgendo".


Nota 1)

Anche a me il termine in (z')^2 piace


Nota 2)


Nel problema della pallina la soluzione apparentemente non e' reversibile nel tempo (per vederlo considerare pallina all'istante di massima compressione e fare andare il tempo in avanti o all'indietro). Non so se diventa di nuovo reversibile se si conta il movimento della Terra.


Nota 3)



Se nei problemi d'urto il moto di tutto il sistema si potesse sempre ricavare senza considerare i dettagli dell'interazione, allora non si potrebbero usare gli esperimenti d'urto per sapere qualcosa sull'interazione. Naturalmente nel caso della che stiamo esaminando interessa solo il moto della parte "grande" del sistema, ma provo ad indovinare che l'idea "i dettagli contano" si applichi anche per questa parte della soluzione.

Wakinian Tanka

unread,
Aug 20, 2019, 11:20:02 AM8/20/19
to
Il giorno martedì 20 agosto 2019 08:05:03 UTC+2, Carlo Pierini ha scritto:
...
> Intuitivamente si capisce che, se fino all’arresto dell’estremo A, la
> quantità di moto dell’anello era p=mv, l’arresto di A dovrà comportare
> necessariamente un aumento di velocità di B, visto che m e p devono restare
> costanti;


Falso. La quantita' di moto NON si conserva durante l'arresto dell'estremo A dell'anello, visto che su esso agisce una forza ESTERNA a risultante non nulla durante quel processo: la reazione vincolare dell'anello fermo.

Lo hai scritto anche nell'altro thread relativamente alla sbarra che cade, e li ti avevo gia' fatto notare che era errato.

--
Wakinian Tanka

Soviet_Mario

unread,
Aug 20, 2019, 12:00:03 PM8/20/19
to
Il 20/08/19 16:59, Wakinian Tanka ha scritto:
scusa, ma dire che non si conserva significa che ci sono
delle perdite
però non equivale a dire che una parte della quantità di
moto può essere comunque trasferita. Per poter dire questo
questa forza esterna a risultante non nulla dovrebbe
cancellare esattamente la quantità di moto della caduta.
E' così ? Come mai ?

Giorgio Bibbiani

unread,
Aug 20, 2019, 12:06:02 PM8/20/19
to
Il 20/08/2019 14.39, JTS ha scritto:
...
> Il problema e' il seguente: c'e' una piastra pesante che cade
> verticalmente su una pallina elastica molto piu' leggera poggiata per
> terra. Supponiamo che nell'urto non venga dissipata energia (la quale
> pero' potrebbe essere dissipata dopo): a che altezza salta la
> pallina?

Nel seguito suppongo di poter trascurare tutti
gli effetti dissipativi, altrimenti ovviamente
il problema della apparente violazione del
principio di reversibilità meccanica del moto
non si porrebbe neppure.

> La soluzione si trova ammettendo che finche' piastra e pallina sono a
> contatto la velocita' di ogni punto della pallina sia lineare con la
> distanza dal suolo. In questo modo si trova che la velocita' del
> centro di massa della pallina e' uguale a meta' della velocita' della
> piastra e che la pallina salta ad un quarto dell'altezza da cui e'
> caduta la piastra.
>

OK, sottolineo che appunto il risultato (approssimativo,
la piastra non potrebbe risalire alla stessa quota
che aveva inizialmente se la pallina avesse massa non
nulla) dipende da quell'ipotesi sulla velocità dei vari
punti della molla.

> L'ipotesi che la velocita' della piastra quando la pallina ha di
> nuovo la forma iniziale (dopo la compressione ed espansione) sia
> uguale alla velocita' al primo contatto la possiamo considerare
> "generale", ma l'ipotesi che la velocita' sia una funzione lineare
> della posizione no.
...
> Nel problema della pallina la soluzione apparentemente non e'
> reversibile nel tempo (per vederlo considerare pallina all'istante di
> massima compressione e fare andare il tempo in avanti o
> all'indietro). Non so se diventa di nuovo reversibile se si conta il
> movimento della Terra.

Direi che il movimento della Terra sia
ininfluente con ottima approssimazione ;-).

Consideriamo prima una pallina (d'ora in avanti molla,
per sottolinearne l'elasticità) ideale di massa nulla,
allora il problema della reversibilità non si
pone perché dal punto di vista meccanico la molla
in quiete equivale a quella in moto, in questo caso
la "molla" è solo una particolare forza esterna
dipendente dalla posizione agente sulla piastra
e vale il principio di reversibilità del moto.

Consideriamo ora una molla di massa m piccola
rispetto a quella della piastra, ma non nulla:

a) quando la piastra in caduta con velocità v
urta la molla allora in base al modello sopra
le varie parti della molla "istantaneamente"
acquistano una velocità che varia da 0 a v
in modo lineare con la quota, ma ciò significa
che la quantità di moto della molla è variata
"istantaneamente", cioè la piastra, nell'urto,
ha esercitato una forza impulsiva F sulla molla

b) quando la molla si espande e raggiunge
la sua lunghezza iniziale con velocità v
dell'estremità superiore, la piastra ha la
stessa velocità v, e la piastra e la molla
si separano senza che queste velocità
varino impulsivamente, cioé la forza F
non c'è più, ma se le forze sono diverse
nella compressione e nell'allungamento
della molla allora non è applicabile il
principio di invertibilità del moto...

Abbiamo però un'apparente incoerenza,
forze diverse nella compressione e nella
espansione della molla, come se ne esce?!

Se ne esce ricordandoci qualcosa che
avevamo trascurato nel nostro modello
ingenuo: la molla massiva non può
trasmettere istantaneamente le forze,
quando la piastra urta inizialmente
la molla produce in questa un'onda di
compressione che si propaga nella
molla con una velocità finita, anche
nella molla compressa al massimo e
durante la fase di espansione
saranno presenti delle onde, ed è
proprio la presenza di queste onde
nella configurazione di massima
compressione della molla che rende
non applicabile l'inversione temporale:
perché questa valesse occorrerebbe
che si invertissero anche le velocità
di propagazione delle varie onde
presenti nella molla pur quando questa
è compressa al massimo.

Ciao

--
Giorgio Bibbiani
(mail non letta)

Wakinian Tanka

unread,
Aug 20, 2019, 5:20:02 PM8/20/19
to
Il giorno martedì 20 agosto 2019 18:00:03 UTC+2, Soviet_Mario ha scritto:
> Wakinian Tanka wrote [rispondendo a Pierinic]:
...
> > Falso. La quantita' di moto NON si conserva durante l'arresto dell'estremo
> > A dell'anello, visto che su esso agisce una forza ESTERNA a risultante non
> > nulla durante quel processo: la reazione vincolare dell'anello fermo.
> > Lo hai scritto anche nell'altro thread relativamente alla sbarra che cade,
> > e li ti avevo gia' fatto notare che era errato.
>
> scusa, ma dire che non si conserva significa che ci sono
> delle perdite
>
Di quantita' di moto (qdm)? Non si parla di "perdite" in questo caso perche' la qdm "persa" non necessariamente si trasferisce ad un altro corpo.



Riprendiamo l'esempio di Elio Fabri nel thread correlato (sulla sbarra che cade): un blocco solido, di massa m, che si muove ad una certa velocita' v e collide su un ostacolo fisso; facciamo ad es il caso di un blocco di piombo (non eccessivamente veloce) che striscia senza attrito su un pavimento orizzontale e poi collide su una rigida parete di pietra verticale (un muro) fermandosi su essa; la quantita' di moto iniziale, m•v, del blocco scompare dopo l'urto, ma non viene acquisita dal muro in pietra, che non si sposta di un micron.
Comunque e' questione di terminologia, credo.

> però non equivale a dire che una parte della quantità di
> moto può essere comunque trasferita.
>
Esatto, come ho scritto sopra. Nel senso: una parte della qdm "puo', in generale" essere trasferita, ma puo' anche non esserlo.
>
> Per poter dire questo
>
Cosa, che la qdm persa dal corpo viene acquisita da un altro corpo? Oppure cosa intendi?
>
> questa forza esterna a risultante non nulla dovrebbe
> cancellare esattamente la quantità di moto della caduta.
> E' così ? Come mai ?
>
Qui non ho proprio capito cosa chiedi.
Ciao.

--
Wakinian Tanka

Carlo Pierini

unread,
Aug 20, 2019, 5:20:02 PM8/20/19
to
Il giorno martedì 20 agosto 2019 17:20:02 UTC+2, Wakinian Tanka ha scritto:
> Il giorno martedì 20 agosto 2019 08:05:03 UTC+2, Carlo Pierini ha scritto:
> ...
> > Intuitivamente si capisce che, se fino all’arresto dell’estremo A, la
> > quantità di moto dell’anello era p=mv, l’arresto di A dovrà comportare
> > necessariamente un aumento di velocità di B, visto che m e p devono restare
> > costanti;

> WAKINIAN

> Falso. La quantita' di moto NON si conserva durante l'arresto dell'estremo A dell'anello, visto che su esso agisce una forza ESTERNA a risultante non nulla durante quel processo: la reazione vincolare dell'anello fermo.
> Lo hai scritto anche nell'altro thread relativamente alla sbarra che cade, e li ti avevo gia' fatto notare che era errato.

CARLO






Il mio era un discorso volutamente approssimato per far capire che, sebbene l'estremo A di ciascun anello venga decelerato da quella forza esterna fino all'arresto, l'altro estremo B aumenta la sua accelerazione a valori superiori a g e la trasmette agli anelli successivi in "cascata" fino ad accrescere finalmente l'accelerazione del manubrio (colpo di frusta). In altri termini, sebbene una certa parte dell'energia cinetica (e quindi della quantità di moto) venga "decurtata" da quella forza (lavoro negativo), una buona parte di entrambe (Ec e p) SI CONSERVA dopo l'arresto di A e viene ceduta all'anello successivo, che incrementa così la sua accelerazione, ...e così via in sequenza per tutti gli anelli fino al manubrio, la cui accelerazione crescerà esponenzialmente a partire dal valore g iniziale. Tale incremento di accelerazione sarà, naturalmente, tanto più alto quanto più alto sarà il valore della massa della catena.




Soviet_Mario

unread,
Aug 20, 2019, 8:54:02 PM8/20/19
to
Il 20/08/19 18:57, Wakinian Tanka ha scritto:
> Il giorno martedì 20 agosto 2019 18:00:03 UTC+2, Soviet_Mario ha scritto:
>> Wakinian Tanka wrote [rispondendo a Pierinic]:
> ...
>>> Falso. La quantita' di moto NON si conserva durante l'arresto dell'estremo
>>> A dell'anello, visto che su esso agisce una forza ESTERNA a risultante non
>>> nulla durante quel processo: la reazione vincolare dell'anello fermo.
>>> Lo hai scritto anche nell'altro thread relativamente alla sbarra che cade,
>>> e li ti avevo gia' fatto notare che era errato.
>>
>> scusa, ma dire che non si conserva significa che ci sono
>> delle perdite
>>

> Di quantita' di moto (qdm)? Non si parla di "perdite" in questo caso perche' la qdm "persa" non necessariamente si trasferisce ad un altro corpo.
>
>
>



> Riprendiamo l'esempio di Elio Fabri nel thread correlato (sulla sbarra che cade): un blocco solido, di massa m, che si muove ad una certa velocita' v e collide su un ostacolo fisso; facciamo ad es il caso di un blocco di piombo (non eccessivamente veloce) che striscia senza attrito su un pavimento orizzontale e poi collide su una rigida parete di pietra verticale (un muro) fermandosi su essa; la quantita' di moto iniziale, m•v, del blocco scompare dopo l'urto, ma non viene acquisita dal muro in pietra, che non si sposta di un micron.
> Comunque e' questione di terminologia, credo.

non credo.

Sto dicendo che tra la situazione totalmente elastica e
quest'ultimo esempio che citi che mi pare totalmente
ANELASTICO (quindi la qdm svanisce integralmente) ci possono
essere un'infinità di sfumature intermedie

>
>> però non equivale a dire che una parte della quantità di
>> moto può essere comunque trasferita.
>>
> Esatto, come ho scritto sopra. Nel senso: una parte della qdm "puo', in generale" essere trasferita, ma puo' anche non esserlo.

ah ... allora non avevo inteso bene io

>>
>> Per poter dire questo
>>
> Cosa, che la qdm persa dal corpo viene acquisita da un altro corpo? Oppure cosa intendi?
>>
>> questa forza esterna a risultante non nulla dovrebbe
>> cancellare esattamente la quantità di moto della caduta.
>> E' così ? Come mai ?
>>
> Qui non ho proprio capito cosa chiedi.

non importa ... ci sto capendo piuttosto poco in questo 3D,
è al di sopra dei miei mezzi (e tempo) :\


> Ciao.
>
> --
> Wakinian Tanka

Wakinian Tanka

unread,
Aug 21, 2019, 5:50:02 AM8/21/19
to
Il giorno mercoledì 21 agosto 2019 02:54:02 UTC+2, Soviet_Mario ha scritto:
> Il 20/08/19 18:57, Wakinian Tanka ha scritto:
>
> > Comunque e' questione di terminologia, credo.
>
> non credo.
> Sto dicendo che tra la situazione totalmente elastica e
> quest'ultimo esempio che citi che mi pare totalmente
> ANELASTICO (quindi la qdm svanisce integralmente) ci possono
> essere un'infinità di sfumature intermedie
>
Ma i termini "elastico" "anelastico" si riferiscono alla conservazione /dell'energia meccanica/ e non della quantita' di moto!

La qdm potrebbe conservarsi ma l'energia meccanica no, o viceversa; per esempio nel primo caso l'urto sarebbe anelastico con conservazione della qdm.
Forse era questo che ti mancava?

--
Wakinian Tanka

Soviet_Mario

unread,
Aug 21, 2019, 8:24:02 AM8/21/19
to
Il 21/08/19 10:43, Wakinian Tanka ha scritto:
hai ragione : che svarione epocale !
Epic Fail :(

MM

unread,
Aug 22, 2019, 7:15:03 AM8/22/19
to
On Thu, 15 Aug 2019 08:46:29 -0700, Wakinian Tanka wrote:

> Due uguali manubri da ginnastica di massa m = 1,5 kg vengono fatti
> cadere simultaneamente da una altezza di 10 m, in aria.
> Uno dei due pero' e' legato ad una lunga catena metallica (piu' di 10m)
> la cui altra estremita' e' legata ad una mensola posta alla stessa
> altezza da cui i manubri sono fatti cadere, e vicino ad esso (manubrio)
> come in figura:
> https://i.postimg.cc/sDC5nv5R/IMG-20190815-172002.jpg La lunghezza
> della catena e' piu' che sufficiente per permettere al secondo manubrio
> di toccare il suolo.
>
> I due manubri toccano il suolo simultaneamente (per quanto si puo'
> apprezzare ad occhio nudo)?
> Uno dei due tocca il suolo prima dell'altro?
>
> Motivare la risposta.

Ho letto il thread solo ora sulla catena, rispondo qua.
Non mi sembra tanto difficile. Arriva prima il manubrio legato
alla catena. (trascuriamo attritti ecc.)
Essendo infatti questo un sistema piu' pesante, ha maggiore energia.
Man mano che scende, "perde" massa. Verso la fine resta
solo il manubrio, che "usufruisce" di questo eccesso di energia
per aumentare la propria velocita'.

Si potrebbe obiettare: ma corpi piu' pesanti o leggeri cadono alla
stessa maniera, trascurando l'attrito con l'aria.
Vero, ma non e' questo il caso:
all'inizio del volo ho:
E=mgh
Alla fine del volo ho:
E = mv^2/2

La m e' la stessa. Nel nostro caso abbiamo rispettivamente
M ed m, con magari M >> m .

MM

unread,
Aug 22, 2019, 9:50:03 AM8/22/19
to
Quantifichiamo.
Sia x lo spostamento in verticale. Abbiamo dunque la massa in funzione
dello spazio percorso, m(x),
con m(0) = M (massa manubrio + massa mezza catena, diciamo)
m(h) = m (solo massa manubrio)
e decresce linearmente in x:
m(x) = M - x(M-m)/h

mentre la velocita' e' data dalla conservazione della energia

v(x)^2 = 2 gh M/m(x)

Dato che m(x) e' sempre minore di M dopo la partenza, la velocita'
e' sempre superiore alla velocita' del solo manubrio, costante e data da
v^2 = 2gh

sempre trascurando l'attrito dell'aria e gli attriti della catena.

Se lo volessi in funzione del tempo, chiaramente va sostituito
x(t) = t v(t)
e bisogna risolvere l'equazione ricorsiva, con v(0) = 0.

Elio Fabri

unread,
Aug 22, 2019, 10:12:02 AM8/22/19
to
MM ha scritto:
> Non mi sembra tanto difficile. Arriva prima il manubrio legato
> alla catena. (trascuriamo attritti ecc.)
> Essendo infatti questo un sistema piu' pesante, ha maggiore energia.
> Man mano che scende, "perde" massa. Verso la fine resta
> solo il manubrio, che "usufruisce" di questo eccesso di energia
> per aumentare la propria velocita'.
Stai dando per sconato che la parte di catena legata a un supporto
fisso rimanga in quiete.
Ossia che quando un anello passa dalla parte che cade a quella che sta
ferma, perde la sua energia cinetica cedendola al resto di
catena+manubrio.
E questo senza alcuna dissipazione (per es. causa urti anelastici).

Ma lo devi dimostrare.
Io non credo sia vero, anche se non ho la soluzione.


--
Elio Fabri

JTS

unread,
Aug 22, 2019, 11:50:03 AM8/22/19
to
On Thursday, August 22, 2019 at 3:50:03 PM UTC+2, MM wrote:

>
> mentre la velocita' e' data dalla conservazione della energia
>

Come fai ad essere sicuro che l'energia si conservi? Ci possono essere dei processi di dissipazione obbligati - date le forze e i vincoli puo' essere che nella catena si debbano per forza generare delle onde.

MM

unread,
Aug 22, 2019, 11:50:03 AM8/22/19
to
On Thu, 22 Aug 2019 16:09:57 +0200, Elio Fabri wrote:

> MM ha scritto:
>> Non mi sembra tanto difficile. Arriva prima il manubrio legato alla
>> catena. (trascuriamo attritti ecc.)
>> Essendo infatti questo un sistema piu' pesante, ha maggiore energia.
>> Man mano che scende, "perde" massa. Verso la fine resta solo il
>> manubrio, che "usufruisce" di questo eccesso di energia per aumentare
>> la propria velocita'.
> Stai dando per sconato che la parte di catena legata a un supporto fisso
> rimanga in quiete.
> Ossia che quando un anello passa dalla parte che cade a quella che sta
> ferma, perde la sua energia cinetica cedendola al resto di
> catena+manubrio.
> E questo senza alcuna dissipazione (per es. causa urti anelastici).
>
Le dissipazioni, come ti dicevo, le trascuro altrimenti non ne usciamo
vivi. Bisognerebbe provare con varie catene e vari pesi per avere
conferme. Anche una fune d'acciaio sarebbe forse meglio.
La dinamica delle catene e' decisamente un problema difficile da trattare
per bene. Essendo gli anelli in acciaio si presume non ci siano grosse
energie negli urti anelastici. Fosse gomma.
Da quel che so sui cavi metallici, gia' il fatto di avere parti
lubrificate o meno cambia in maniera drastica il comportamento sotto
carico, chiedi ai progettisti di funivie. Non hanno modelli rigorosi,
vanno per esperienza e misure.
Chi ha studiato per bene le onde nelle catene? Forse qualche progettista
navale per le ancore, non so.

Del resto il mio Ansatz funziona, i fatti mostrano che arriva prima,
quindi in fine ha piu' energia a disposizione. E quel che dissipa in vari
modi non basta a rallentarla tanto.
> Ma lo devi dimostrare.
> Io non credo sia vero, anche se non ho la soluzione.
Spiegati meglio. A parte le approssimazioni di cui parlavo a cosa ti
riferisci? Attrito con l'aria, dissipazione fra anelli catena, catena
decisamente piu' rigida di una fune, catena di lunghezza variabile per il
gioco fra anelli.


In prima approssimazione considero una catena di lunghezza L appena piu'
grande dell'altezza (L=h+ epsilon) per semplificarci la vita.
La prima meta' resta ferma, il resto perde massa come descrivevo.
Naturalmente non e' proprio cosi', si genera anche un'onda nella catena
che combinera' qualche altro scherzo.
L'unica sarebbe risolvere d'Alembert numericamente, aggiungere termini
dissipativi e vedere quando si incontra il punto critico per cui il peso
sulla catena e' piu' lento.

Mi sovviene un caso piu' semplice, senza onde di sorta.
Un sacco di sabbia bucato lateralmente (altrimenti...) vs sacco di sabbia
integro. A favore di quello integro minore perdita per attrito con aria,
a favore di quello bucato maggiore velocita' per massa che si riduce.
Chi vincera'?

Wakinian Tanka

unread,
Aug 22, 2019, 1:15:03 PM8/22/19
to
Anche senza onde, l'energia non si conserva, basta guardare la dimostrazione che ho fatto nel caso della sbarra che cade ed urta il vincolo in A, nel thread  omonimo "sbarra che cade e si blocca ad un estremo":

Pag1;
https://i.postimg.cc/G26PWxf5/IMG-20190819-165937.jpg

Pag2:
https://i.postimg.cc/9fRr6Hjc/IMG-20190819-165901.jpg

--
Wakinian Tanka

MM

unread,
Aug 22, 2019, 1:20:02 PM8/22/19
to
On Thu, 22 Aug 2019 06:53:15 -0700, JTS wrote:


>> mentre la velocita' e' data dalla conservazione della energia
>>
>>
> Come fai ad essere sicuro che l'energia si conservi?
Cito riga 3:
(trascuriamo attritti ecc.)

sarebbe "attriti".
Inoltre, pur con gli attriti funziona come dico io, arriva prima e si
vede chiaramente.
Se vuoi studiare il caso completo con tutte le dissipazioni nulla in
contrario. Ovviamente sara' una roba numerica. Si puo' fare con blender
ad esempio.

JTS

unread,
Aug 22, 2019, 2:20:02 PM8/22/19
to
Am 22.08.2019 um 19:07 schrieb MM:
> On Thu, 22 Aug 2019 06:53:15 -0700, JTS wrote:
>
>
>>> mentre la velocita' e' data dalla conservazione della energia
>>>
>>>
>> Come fai ad essere sicuro che l'energia si conservi?
> Cito riga 3:
> (trascuriamo attritti ecc.)
>
> sarebbe "attriti".


Non penso la dissipazione minima dipenda dagli attriti, ma dai vincoli
che inducono obbligatoriamente vibrazioni. Il caso della sbarra che
viene improvvisamente bloccata (vedi post di Wakinian Tanka) e' un buon
esempio: non ci sono attriti, ma l'energia non si conserva. Questo
significa che ci devono essere per forza vibrazioni, anche se tu non le
hai "messe dentro il modello".

JTS

unread,
Aug 22, 2019, 2:20:02 PM8/22/19
to
Am 22.08.2019 um 19:05 schrieb Wakinian Tanka:
> Il giorno giovedì 22 agosto 2019 17:50:03 UTC+2, JTS ha scritto:
>> On Thursday, August 22, 2019 at 3:50:03 PM UTC+2, MM wrote:
>>
>>> mentre la velocita' e' data dalla conservazione della energia
>>
>> Come fai ad essere sicuro che l'energia si conservi? Ci possono essere dei
>> processi di dissipazione obbligati - date le forze e i vincoli puo' essere
>> che nella catena si debbano per forza generare delle onde.
>
>
> Anche senza onde, l'energia non si conserva

Dove va?

Wakinian Tanka

unread,
Aug 22, 2019, 2:20:03 PM8/22/19
to
Il giorno giovedì 22 agosto 2019 13:15:03 UTC+2, MM ha scritto:
...
> Ho letto il thread solo ora sulla catena, rispondo qua.
> Non mi sembra tanto difficile. Arriva prima il manubrio legato
> alla catena. (trascuriamo attritti ecc.)
> Essendo infatti questo un sistema piu' pesante, ha maggiore energia.
> Man mano che scende, "perde" massa. Verso la fine resta
> solo il manubrio, che "usufruisce" di questo eccesso di energia
> per aumentare la propria velocita'.
>

Be', ma questo stesso ragionamento di potrebbe applicare anche ad un razzo che inizialmente di muove a velocita' v e poi aziona il motore "in frenamento" cioe' espellendo i gas nel senso di marcia: la massa del razzo diminuisce ma diminuisce anche la sua energia cinetica...
Non basta dire "la massa diminuisce".
Tutto dipende da come si comporta la massa espulsa (vedi al riguardo post di JTS e di Giorgio Pastore).

--
Wakinian Tanka

MM

unread,
Aug 22, 2019, 4:15:02 PM8/22/19
to
On Thu, 22 Aug 2019 06:53:15 -0700, JTS wrote:

> On Thursday, August 22, 2019 at 3:50:03 PM UTC+2, MM wrote:
>
>
>> mentre la velocita' e' data dalla conservazione della energia
>>
>>
> Come fai ad essere sicuro che l'energia si conservi?

Un sistema con vincoli olonomi conserva l'energia.
https://is.gd/yuEFHK

Cio' avviene anche per alcune classi di vincoli anolonomi
https://is.gd/qXjaO1
https://is.gd/0ItxEb

MM

unread,
Aug 22, 2019, 4:15:02 PM8/22/19
to
On Thu, 22 Aug 2019 10:51:56 -0700, Wakinian Tanka wrote:


> Non basta dire "la massa diminuisce".
Difatti ho messo i conti in un post successivo.

Wakinian Tanka

unread,
Aug 22, 2019, 4:15:03 PM8/22/19
to
Tu con "onde" intendevi anche "vibrazioni dei singoli anelli"? Allora ok. Pensavo che intendessi "onde trasversali di tutta la catena".

--
Wakinian Tanka

JTS

unread,
Aug 22, 2019, 4:30:02 PM8/22/19
to
Non avevo pensato alla distinzione :-)

JTS

unread,
Aug 22, 2019, 4:55:03 PM8/22/19
to
Questo potrebbe essere un sistema con vincoli anolonomi di quelli che
non la conservano. Ogni pezzo di catena non e' soggetto al vincolo fino
a quando non diviene parte della parte orizzontale. In quel momento
compare il vincolo che blocca un estremo. La schematizzazione come
sbarra che cade e si blocca ad un estremo mi pare un buon inizio, e in
quel caso il calcolo (semplice) mostra che l'energia non si conserva.

Carlo Pierini

unread,
Aug 22, 2019, 5:25:02 PM8/22/19
to

Riflessione finale (più accurata).








Consideriamo la sbarra in caduta libera citata nell’altro thread “Sbarra che cade e si blocca…”). Immaginiamo di bloccare l’estremo A imperniandolo rigidamente ad un vincolo solidale col …pianeta Terra: la sbarra eserciterà sul vincolo una forza che, per quanto sia alto il suo valore F, compirà un lavoro L = F*s che sarà di valore infinitesimale essendo “s” lo spostamento infinitesimale che F produce …sul pianeta Terra (considerando trascurabile lo spostamento per deformazione - elastica e/o permanente - del vincolo). Questo significa, cioè, che l’energia cinetica (Ec) e la qdm=mv acquisite dalla sbarra fino al momento dell’arresto dell’estremità A resteranno pressoché invariate; pertanto tale arresto comporterà necessariamente un aumento dell’accelerazione dell’estremità B, visto che m e p devono restare costanti (o quasi); in altre parole, la velocità perduta dal lato A deve essere acquistata dal lato B, se vogliamo che l’accelerazione media sia ancora <g>.

Ora, riportiamo questo ragionamento al caso del manubrio e della catena:

https://i.postimg.cc/D0nm32NW/IMG-20190822-0003.jpg






Supponiamo che i segmenti a-b-c-d-e siano gli anelli della catena e che ciascuno di essi sia assimilabile alla sbarra in caduta libera di cui sopra. Quando, durante la caduta libera, l’anello <d> soggetto all’accelerazione media <g> si sarà portato nella posizione <e> e il suo estremo A sarà arrestato dal perno solidale col …pianeta Terra, il suo estremo B assumerà una accelerazione superiore a <g> e la trasmetterà (mutatis mutandis) agli anelli successivi c-b-a e quindi al manubrio <M>. Cosicché, ripetendosi il medesimo processo per gli anelli successivi (c-b-a), gli incrementi di accelerazione si sommeranno (in un effetto frusta) e trascineranno il manubrio in una accelerazione via via crescente fino al contatto col suolo.

In definitiva, è l’energia cinetica della metà della catena in caduta libera quella che sarà in gran parte trasferita, anello dopo anello, al manubrio facendogli toccare il suolo per primo.

Wakinian Tanka

unread,
Aug 23, 2019, 8:15:02 AM8/23/19
to
Il giorno giovedì 22 agosto 2019 22:15:02 UTC+2, MM ha scritto:

> Un sistema con vincoli olonomi conserva l'energia.
> https://is.gd/yuEFHK
>
Guarda che l'energia si conserva SEMPRE, per sistemi isolati, che sia meccanica, elettromagnetica, nucleare o quel che preferisci.
Qui stiamo parlando *dell'energia cinetica dell'asta*.

Dimostrami che in un vincolo che appare all'improvviso si conserva l'energia cinetica del corpo che viene vincolato. Non puoi. Ti faccio un esempio banale: l'asta viene vincolata all'improvviso non in A ma nel cdm: l'energia cinetica dell'asta sparisce totalmente...

Wakinian Tanka:
> > Non basta dire "la massa diminuisce".

MM:
> Difatti ho messo i conti in un post successivo

I tuoi conti sono relativi al ragionamento sbagliato quindi sono per forza sbagliati (ovvero: se non li facevi risparmiavi tempo).

MM

unread,
Aug 23, 2019, 9:45:02 AM8/23/19
to
On Fri, 23 Aug 2019 01:36:19 -0700, Wakinian Tanka wrote:

> Il giorno giovedì 22 agosto 2019 22:15:02 UTC+2, MM ha scritto:
>
>> Un sistema con vincoli olonomi conserva l'energia.
>> https://is.gd/yuEFHK
>>
> Guarda che l'energia si conserva SEMPRE, per sistemi isolati, che sia
> meccanica, elettromagnetica, nucleare o quel che preferisci.
> Qui stiamo parlando *dell'energia cinetica dell'asta*.
>
Un vincolo anolonomo non conserva necessariamente energia, vedi dopo.
Per la conservazione dell'energia il sistema deve essere invariante per
traslazioni temporali. Spesso e' l'unico integrale primo del sistema.

> Dimostrami che in un vincolo che appare all'improvviso si conserva
> l'energia cinetica del corpo che viene vincolato. Non puoi. Ti faccio un
> esempio banale: l'asta viene vincolata all'improvviso non in A ma nel
> cdm: l'energia cinetica dell'asta sparisce totalmente...
>

Un vincolo che appare all'improvviso NON E' per definizione olonomo in
quanto come dici tu stesso dipende dal tempo.
Il tuo ragionamento non regge.

> Wakinian Tanka:
>> > Non basta dire "la massa diminuisce".
>
> MM:
>> Difatti ho messo i conti in un post successivo
>
> I tuoi conti sono relativi al ragionamento sbagliato quindi sono per
> forza sbagliati (ovvero: se non li facevi risparmiavi tempo).
Dunque non e' vero che cambia la massa. Interessante.
Mostrami i conti giusti e le conclusioni giuste allora, io non li vedo.

JTS

unread,
Aug 23, 2019, 10:35:02 AM8/23/19
to
Am 22.08.2019 um 20:35 schrieb MM:

MM

unread,
Aug 23, 2019, 10:40:02 AM8/23/19
to
On Thu, 22 Aug 2019 22:51:47 +0200, JTS wrote:

> Am 22.08.2019 um 20:35 schrieb MM:
>> On Thu, 22 Aug 2019 06:53:15 -0700, JTS wrote:
>>
>>> On Thursday, August 22, 2019 at 3:50:03 PM UTC+2, MM wrote:
>>>
>>>
>>>> mentre la velocita' e' data dalla conservazione della energia
>>>>
>>>>
>>> Come fai ad essere sicuro che l'energia si conservi?
>>
>> Un sistema con vincoli olonomi conserva l'energia. https://is.gd/yuEFHK
>>
>> Cio' avviene anche per alcune classi di vincoli anolonomi
>> https://is.gd/qXjaO1 https://is.gd/0ItxEb
>>
>>
> Questo potrebbe essere un sistema con vincoli anolonomi di quelli che
> non la conservano.

Non dipende dalla velocita' ne' dal tempo. Perche' anolonomo? Spiegati.

>Ogni pezzo di catena non e' soggetto al vincolo fino
> a quando non diviene parte della parte orizzontale. In quel momento
> compare il vincolo che blocca un estremo.

Ma non impedisce ad ogni anello di ruotare. Se getto una palla sul
pavimento quella rimbalza indietro, non dissipa energia in prima
approssimazione.

Il punto e' in realta' inessenziale, chiaramente il sistema dissipa
energia in qualche modo. Bisogna dimostrare che, calcoli alla mano, il
peso con catena arriva giu' per primo. Ovvero, come ho scritto prima

v(x)^2 = 2*g*h*M/m(x)

con M>m(x) .

Oppure con una bella simulazione, ribadisco.

Giorgio Pastore

unread,
Aug 23, 2019, 10:40:02 AM8/23/19
to
Il 23/08/19 10:36, Wakinian Tanka ha scritto:
> Il giorno giovedì 22 agosto 2019 22:15:02 UTC+2, MM ha scritto:
>
>> Un sistema con vincoli olonomi conserva l'energia.
>> https://is.gd/yuEFHK
>>
> Guarda che l'energia si conserva SEMPRE, per sistemi isolati, che sia meccanica, elettromagnetica, nucleare o quel che preferisci.
> Qui stiamo parlando *dell'energia cinetica dell'asta*.
>
> Dimostrami che in un vincolo che appare all'improvviso si conserva l'energia cinetica del corpo che viene vincolato. Non puoi. Ti faccio un esempio banale: l'asta viene vincolata all'improvviso non in A ma nel cdm: l'energia cinetica dell'asta sparisce totalmente...


Ma perché non te la prendi con più calma evitando di "aggredire" chi
partecipa alle discussioni del NG? Devi dimostrare qualcosa?

Un urto elastico lo puoi tranquillamente vedere come un vincolo "che
appare all' improvviso".

Inoltre MM parlava chiaramente di energia meccanica. che non ha nessun
obbligo di conservarsi.

> Wakinian Tanka:
>>> Non basta dire "la massa diminuisce".
>
> MM:
>> Difatti ho messo i conti in un post successivo
>
> I tuoi conti sono relativi al ragionamento sbagliato quindi sono per forza sbagliati (ovvero: se non li facevi risparmiavi tempo).
>

Magari discutere *dove* sarebbero sbagliati secondo te, sarebbe più
utile. Ti faccio anhe presente che "ex falso quodlibet", quindi la tua
affermazione che derivando da un ragionamento sbagliato debbano essere
sbagliati va contro la logica dell' implicazione.

Carlo Pierini

unread,
Aug 23, 2019, 10:45:02 AM8/23/19
to
Il giorno venerdì 23 agosto 2019 14:15:02 UTC+2, Wakinian Tanka ha scritto:


> Dimostrami che in un vincolo che appare all'improvviso si conserva l'energia cinetica del corpo che viene vincolato. Non puoi. Ti faccio un esempio banale: l'asta viene vincolata all'improvviso non in A ma nel cdm: l'energia cinetica dell'asta sparisce totalmente...

WAKINIAN

Dimostrami che in un vincolo che appare all'improvviso si conserva l'energia cinetica del corpo che viene vincolato. Non puoi. Ti faccio un esempio banale: l'asta viene vincolata all'improvviso non in A ma nel cdm: l'energia cinetica dell'asta sparisce totalmente...

CARLO




Una cosa è certa: nel momento in cui l’estremo A di ciascun anello in caduta libera resta vincolato, SOLO SE l’estremo B aumenta la sua accelerazione a valori superiori a <g> può realizzarsi quel trasferimento di energia cinetica dagli anelli al manubrio che è indispensabile affinché il manubrio stesso accresca la propria accelerazione verso il basso. Se ciò non accadesse (cioè, se l’estremo B conservasse la sua accelerazione naturale <g>), il nostro manubrio toccherebbero il suolo nel medesimo istante dell’altro (trascurando gli attriti), poiché NULLA accrescerebbe la sua accelerazione.



Naturalmente, il calcolo di questo incremento di accelerazione di B è impossibile se con conosciamo nel dettaglio le proprietà meccaniche del vincolo e degli anelli della catena (elasticità, massa, durezza, ecc.); ma questa nostra impossibilità non invalida la certezza assoluta che l’estremo B di ciascun anello aumenti la propria accelerazione negli istanti successivi al suo vincolamento e, pertanto, la certezza che gran parte della sua qdm (o della sua Ec) si conservi e si trasferisca all’anello successivo e infine al manubrio.

MM

unread,
Aug 23, 2019, 12:00:03 PM8/23/19
to
On Fri, 23 Aug 2019 14:44:37 +0200, Giorgio Pastore wrote:


>>
> Magari discutere *dove* sarebbero sbagliati secondo te, sarebbe più
> utile. Ti faccio anhe presente che "ex falso quodlibet", quindi la tua
> affermazione che derivando da un ragionamento sbagliato debbano essere
> sbagliati va contro la logica dell' implicazione.

Logica 101, che nessuno mai segue. Approfitto del tuo intervento per
chiederti una cosa, anzi due, se possibile. Sei ferrato in effetti
topologici in fisica? In particolare monopoli magnetici?

Wakinian Tanka

unread,
Aug 23, 2019, 2:40:03 PM8/23/19
to
Il giorno venerdì 23 agosto 2019 16:40:02 UTC+2, Giorgio Pastore ha scritto:
...
> Ma perché non te la prendi con più calma evitando di "aggredire" chi
> partecipa alle discussioni del NG? Devi dimostrare qualcosa?
>

Non ho "aggredito" nessuno, pero', a mio avviso, c'e' qualcuno che pretende di insegnare qualcosa senza rispondere alle contestazioni che gli vengono mosse.
Dove secondo me i conti sarebbero sbagliati e' nei presupposti e l'ho gia' scritto.
MM ha fatto una specie di ragionamento/descrizione/modello, che *a mio parere* e' errato per 2 motivi:
1. l'energia cinetica della catena non si conserva nell'urto.

2. se la catena incrementa la sua velocita' solo perche' perde massa, senza specificare *come* perde massa, allora perche' un razzo che sta frenando il suo moto decelera invece di incrementare la sua velocita'?
Non ho ricevuto una risposta sodfisfacente a 1. e non ho ricevuto alcuna risposta a 2.


Io non devo "dimostrare" nulla, perche' *ho gia'* dimostrato nell'altro thread sulla sbarra che l'energia cinetica non puo' conservarsi nell'urto, e che il suo ragionamento sul fatto che un sistema che perde massa allora incrementa la sua velocita' e' come minimo carente, senza ulteriori precisazioni.
Ora mi pare che tocchi a lui *dimostrare* qualcosa. Non l'ha ancora fatto, mi pare. Tu cosa ne pensi a questo proposito?
Suggerimento (di Fabri): utilizzare la consetvazione del momento della quantita' di moto, dato che l'energia cinetica non si conserva.
L'ho fatto io con un mio modello nel mio post precedente (che a questo punto spero sia gia' stato pubblicato).

--
Wakinian Tanka

Giorgio Pastore

unread,
Aug 23, 2019, 2:40:03 PM8/23/19
to
Il 23/08/19 17:20, MM ha scritto:
....
> Approfitto del tuo intervento per
> chiederti una cosa, anzi due, se possibile. Sei ferrato in effetti
> topologici in fisica? In particolare monopoli magnetici?
>

Molto poco. Non è il mio "pane quotaidiano". Ma, chi sa?

Wakinian Tanka

unread,
Aug 23, 2019, 2:40:03 PM8/23/19
to
Questo e' il mio modello e il modo in cui l'ho "risolto", tra virgolette perche' non so fare l'ultimo integrale a pag. 3


Pag 1:
https://i.postimg.cc/cHLqvvrK/IMG-20190823-174951.jpg

Pag 2:
https://i.postimg.cc/DzyFXjNy/IMG-20190823-181422.jpg

wolphram alpha mi fornisce come soluzione la prima funzione in cima alla pagina; non l'ho verificata:
Pag 3:
https://i.postimg.cc/RVQjL1K3/IMG-20190823-185456.jpg

--
Wakinian Tanka

MM

unread,
Aug 23, 2019, 3:05:02 PM8/23/19
to
OK, proviamo. L'indice z sul prodotto delle due cariche elettriche e
magnetiche i.e. e*g~ z, oltre al solito significato matematico, ha
qualche profondo significato fisico, a parte l'energia di monopolo?

In altre parole, cosa cambia di essenziale al variare di z?

MM

unread,
Aug 23, 2019, 3:35:02 PM8/23/19
to
On Fri, 23 Aug 2019 10:07:17 -0700, Wakinian Tanka wrote:

> Questo e' il mio modello e il modo in cui l'ho "risolto", tra virgolette
> perche' non so fare l'ultimo integrale a pag. 3
>
Si chiamano integrali ellittici e si possono esprimere in termini
della funzione ipergeometrica 2F1.

Esattamente
2F1(1/2,2/3,5/3,x^3)*x^2/2
e' la primitiva di

x/sqrt(1-x^3)

(se leggo bene la tua roba, non si capisce molto).

Poi devi invertirla per ottenere x(t)

Infine confronti il tempo di volo con quello del solo peso e dimostri
che e' inferiore.

Il resto non l'ho controllato, non riesco a leggerlo bene ne' mi convince
cio' che vedo. Parti dalla fine e mostra che ottieni il risultato
corretto.

Wakinian Tanka

unread,
Aug 23, 2019, 5:00:03 PM8/23/19
to
Il giorno venerdì 23 agosto 2019 20:40:03 UTC+2, Wakinian Tanka ha scritto:
> Questo e' il mio modello...


C'era un errore dovuto all'aver dimenticato di moltiplicare per l_0, lunghezza di un singolo anello, nel calcolare il momento della quantita' di moto.

Versione 2.0

Pag 1:
https://i.postimg.cc/3wQPk6mV/IMG-20190823-221611.jpg

Pag 2:
https://i.postimg.cc/C1Q7qB95/IMG-20190823-221944.jpg

Pag 3:
https://i.postimg.cc/zX57Rrcn/IMG-20190823-222149.jpg

Pag 4:
https://i.postimg.cc/nVs5FpQT/IMG-20190823-223652.jpg


Quindi il risultato finale e': se la catena ha n anelli, n >3, allora il rapporto tra la velocita' dell'_ultimo_ anello della porzione di catena che cade, e la velocita' di caduta, dalla stessa altezza, di un punto materiale non vincolato, e'
r = sqrt[(n-1)^3 - 8]/{2•sqrt[3(n-3)]}.

Divertitevi a fare il conto per vari n :-).

--
Wakinian Tanka

Giorgio Pastore

unread,
Aug 23, 2019, 5:25:02 PM8/23/19
to
Il 23/08/19 19:39, Wakinian Tanka ha scritto:
....
> MM ha fatto una specie di ragionamento/descrizione/modello, che *a mio parere* e' errato per 2 motivi:
> 1. l'energia cinetica della catena non si conserva nell'urto.

L'energia cinetica non si conserva mai se c'e' di mezzo una caduta. Devi
tner conto dell' energia potenziale.
>
> 2. se la catena incrementa la sua velocita' solo perche' perde massa, senza specificare *come* perde massa, allora perche' un razzo che sta frenando il suo moto decelera invece di incrementare la sua velocita'?

E' già stato fatto notare (mi semba da JTS, per primo) che nei sistemi a
massa variabile le cose non vanno sempre allo stesso modo. Dipende dalla
velocità della massa "persa". Tra catena e razzo c'e' una differenza
fondamentale proprio li'. Tuttavia la modellizzazione che avevo seguito
inizialmente non è completamente soddisfacente e credo di aver capito
perché; ignora la tensione a livello dell'anello più basso della catena.

> Non ho ricevuto una risposta sodfisfacente a 1. e non ho ricevuto alcuna risposta a 2.

Io mi sono fatto delle idee. Poi, sono anche andato a vedere cosa è già
stato fatto sul problema (moltissimo!).

Trovo importante per la discussione, tener presente il confronto tra
esperimento e modello *in cui si conserva l'energia* in questo lavoro
https://arxiv.org/pdf/1110.6035
De Sousa, C. A., Gordo, P. M., & Costa, P. (2012). Falling chains as
variable-mass systems: Theoretical model and experimental analysis.
European Journal of Physics, 33(4), 1007

In particolare trovo molto interessante la fig.3.

Sospetto che l'aspetto "spiazzante" del problema è che, mentre per la
descrizione completa del sistema occorre tener conto di una non
conservazione dell' energia meccanica nel moto traslazionale della
catena, per la sola parte connessa con il moto verso il basso
dell'estremo libero l'approssimazione di conservazione dell' energia
meccanica sia invece giustificabile.

Scrivo sospetto perché il tempo è poco e per il NG sto già dedicando non
poco dello scarso tempo libero a disposizione per mettere meglio in fila
una volta pe tutte le idee sulla questione delle forze :-(
Spero per la prossima settimana di riuscire a metter fuori qualcosa.

Wakinian Tanka

unread,
Aug 23, 2019, 7:15:02 PM8/23/19
to
Il giorno venerdì 23 agosto 2019 23:25:02 UTC+2, Giorgio Pastore ha scritto:
> Il 23/08/19 19:39, Wakinian Tanka ha scritto:
> ....
> > MM ha fatto una specie di ragionamento/descrizione/modello, che *a mio
> > parere* e' errato per 2 motivi:
> > 1. l'energia cinetica della catena non si conserva nell'urto.
>
> L'energia cinetica non si conserva mai se c'e' di mezzo una caduta. Devi
> tner conto dell' energia potenziale.
>
Giorgio, non a caso ho parlato di "conservazione dell'energia cinetica nell'urto"!
La variazione di energia potenziale durante l'urto e' trascurabile.
>
> Sospetto che l'aspetto "spiazzante" del problema è che, mentre per la
> descrizione completa del sistema occorre tener conto di una non
> conservazione dell' energia meccanica nel moto traslazionale della
> catena, per la sola parte connessa con il moto verso il basso
> dell'estremo libero l'approssimazione di conservazione dell' energia
> meccanica sia invece giustificabile.
>
Perdonami, ma che intendi con "energia meccanica nel moto traslazionale della
catena, per la sola parte connessa con il moto verso il basso
dell'estremo libero", tenuto conto che la porzione di catena che e' in moto diminuisce di massa (e che e' sottoposta ad un campo di forze) ?

--
Wakinian Tanka

MM

unread,
Aug 23, 2019, 7:15:02 PM8/23/19
to
On Fri, 23 Aug 2019 23:14:52 +0200, Giorgio Pastore wrote:


> Io mi sono fatto delle idee. Poi, sono anche andato a vedere cosa è già
> stato fatto sul problema (moltissimo!).
>
> Trovo importante per la discussione, tener presente il confronto tra
> esperimento e modello *in cui si conserva l'energia* in questo lavoro
> https://arxiv.org/pdf/1110.6035 De Sousa, C. A., Gordo, P. M., & Costa,
> P. (2012). Falling chains as variable-mass systems: Theoretical model
> and experimental analysis. European Journal of Physics, 33(4), 1007
>
> In particolare trovo molto interessante la fig.3.
>
> Sospetto che l'aspetto "spiazzante" del problema è che, mentre per la
> descrizione completa del sistema occorre tener conto di una non
> conservazione dell' energia meccanica nel moto traslazionale della
> catena, per la sola parte connessa con il moto verso il basso
> dell'estremo libero l'approssimazione di conservazione dell' energia
> meccanica sia invece giustificabile.

Nelle conclusioni dell'articolo pubblicato su EPJ voll 33 n4 dice
esattamente cio' che affermo, l'energia si conserva, in particolare
troviamo a pg. 20:

As already verified by other authors, the experimental data for the U-
chain shows a good convergence with the energy conserving approach,
independently of the type of chain. There exists continuous interference
(no broken contact) during the motion and, consequently, the dissipative
effects of explicit collisions, even in a discrete chain, are negligible.
In addition,one-dimensional model is a good approach for this
configuration.

L'energia si conserva, i risultati sono in accordo coi dati sperimentali.
Le perdite dissipative per urti fra elementi della catena sono
trascurabili.

Ci sarebbe altro da aggiungere, ma non qua, il thread sta diventando
difficilmente gestibile.

Wakinian Tanka

unread,
Aug 26, 2019, 12:45:03 PM8/26/19
to
Il giorno venerdì 23 agosto 2019 15:45:02 UTC+2, MM ha scritto:
> On Fri, 23 Aug 2019 01:36:19 -0700, Wakinian Tanka wrote:
>
> > Il giorno giovedì 22 agosto 2019 22:15:02 UTC+2, MM ha scritto:
> >
> >> Un sistema con vincoli olonomi conserva l'energia.
> >> https://is.gd/yuEFHK
> >>
> > Guarda che l'energia si conserva SEMPRE, per sistemi isolati, che sia
> > meccanica, elettromagnetica, nucleare o quel che preferisci.
> > Qui stiamo parlando *dell'energia cinetica dell'asta*.
> >
> Un vincolo anolonomo non conserva necessariamente energia,

L'energia si conserva *sempre*.
Solo, si trasforma. In un urto anelastico, ad es, energia meccanica si trasforma in energia interna.
Tu non hai specificato di quale energia stai parlando.
Comunque ho letto anch'io l'articolo di De Sousa e a pag 6 dice:
"A. Energy conserving assumption.
"We start by considering that the mechanical energy of the U-chain is conserved [11, 12, 15]".

Ma il fatto che si conservi "l'energia meccanica" non implica che questa non sia in parte immagazzinata come, ad es, energia potenziale elastica nel metallo. Avrebbe dovuto come minimo precisarlo.

--
Wakinian Tanka

JTS

unread,
Aug 26, 2019, 12:50:03 PM8/26/19
to
Am 22.08.2019 um 23:17 schrieb MM:
> On Thu, 22 Aug 2019 22:51:47 +0200, JTS wrote:
>
>> Am 22.08.2019 um 20:35 schrieb MM:
>>> On Thu, 22 Aug 2019 06:53:15 -0700, JTS wrote:
>>>
>>>> On Thursday, August 22, 2019 at 3:50:03 PM UTC+2, MM wrote:
>>>>
>>>>
>>>>> mentre la velocita' e' data dalla conservazione della energia
>>>>>
>>>>>
>>>> Come fai ad essere sicuro che l'energia si conservi?
>>>
>>> Un sistema con vincoli olonomi conserva l'energia. https://is.gd/yuEFHK
>>>
>>> Cio' avviene anche per alcune classi di vincoli anolonomi
>>> https://is.gd/qXjaO1 https://is.gd/0ItxEb
>>>
>>>
>> Questo potrebbe essere un sistema con vincoli anolonomi di quelli che
>> non la conservano.
>
> Non dipende dalla velocita' ne' dal tempo. Perche' anolonomo? Spiegati.


Per la parte seguente: "Ogni pezzo di catena non e' soggetto al vincolo
fino a quando non diviene parte della parte orizzontale". Puo' essere
che sbagli.

>> Ogni pezzo di catena non e' soggetto al vincolo fino
>> a quando non diviene parte della parte orizzontale. In quel momento
>> compare il vincolo che blocca un estremo.
>
> Ma non impedisce ad ogni anello di ruotare.

Cio' nonostante la dissipazione puo' essere obbligata anche in assenza
di attriti (cfr. problema della sbarra moto di traslazione un cui
estremo viene bloccato)

>
> Il punto e' in realta' inessenziale, chiaramente

Per me e' probabile ma non ne sono certo

> il sistema dissipa
> energia in qualche modo. Bisogna dimostrare che, calcoli alla mano, il
> peso con catena arriva giu' per primo. Ovvero, come ho scritto prima
>
> v(x)^2 = 2*g*h*M/m(x)
>
> con M>m(x) .
>
> Oppure con una bella simulazione, ribadisco.
>

Oppure con la d(mv)/dt = F (che mi sembra un tentativo accettabile).

Piu' semplice: osservando che visto che la catena puo' esercitare solo
tensione, e che il pezzo in caduta "termina" con un tratto orientato
verso il basso, allora c'e' una forza aggiuntiva verso il basso per
qualunque interazione con la parte di catena orizzontale che si sta
fermando (contrariamente a quanto ho detto nel mio primo messaggio in
questo thread!).

MM

unread,
Aug 26, 2019, 3:10:03 PM8/26/19
to
On Fri, 23 Aug 2019 22:22:31 +0200, JTS wrote:


>
>> Il punto e' in realta' inessenziale, chiaramente
>
> Per me e' probabile ma non ne sono certo

E' una certezza. Cito:
https://arxiv.org/pdf/1110.6035

Nelle conclusioni dell'articolo pubblicato su EPJ vol 33 n4 dice
esattamente cio' che affermo, l'energia si conserva, in particolare
troviamo a pg. 20:

As already verified by other authors, the experimental data for the U-
chain shows a good convergence with the energy conserving approach,
independently of the type of chain. There exists continuous interference
(no broken contact) during the motion and, consequently, the dissipative
effects of explicit collisions, even in a discrete chain, are negligible.
In addition,one-dimensional model is a good approach for this
configuration.

L'energia si conserva, i risultati sono in accordo coi dati sperimentali.
Le perdite dissipative per urti fra elementi della catena sono
trascurabili.

Se non sei convinto, puoi scrivere un articolo che dimostri il contrario
e poi farlo pubblicare, meglio ancora se nella stessa rivista.

JTS

unread,
Aug 26, 2019, 3:10:03 PM8/26/19
to
Am 23.08.2019 um 22:22 schrieb JTS:

>>
>
> Oppure con la d(mv)/dt = F (che mi sembra un tentativo accettabile).
>


Adesso che mi sono accorto che con la d(mv)/dt = F l'energia aumenta
(messaggio 26 Agosto 19:16, non ancora approvato in questo momento), il
metodo non mi piace piu' :-)

JTS

unread,
Aug 26, 2019, 3:15:02 PM8/26/19
to
Am 24.08.2019 um 00:49 schrieb MM:

>
> Nelle conclusioni dell'articolo pubblicato su EPJ voll 33 n4 dice
> esattamente cio' che affermo, l'energia si conserva,


Ho confrontato la formula che si ottiene con la d(mv)/dt = F (calcolo di
Giorgio con massa del manubrio = 0,
https://groups.google.com/d/msg/it.scienza.fisica/rq3C80TfCE8/2rr24i2xAAAJ)
e quella che si ottiene con la conservazione dell'energia (formula (7)
di de Sousa et al. https://arxiv.org/pdf/1110.6035.pdf).

Viene fuori che nella formula d(mv)/dt = F l'energia finale della catena
e' superiore all'energia potenziale iniziale, l'energia aumenta ;-)
Me ne sono accorto facendo un grafico delle soluzioni, le equazioni le
ho guardate dopo :-)

Conserv. energia:

x'' = g + (x')^2/(2*(l-x))

d(mv)/dt = F

x'' = g + (x')^2/(l-x)

Dandole per buone (ho fatto i calcoli e mi sono venuti sbagliati) la
ragione e' che se parto con due masse con stessa velocita' e trasferisco
tutta la quantita' di moto da un massa all'altra l'energia del sistema
aumenta.

Scrivo il calcolo qui per fissare quello che intendo

p_iniziale = Mv + mv
E_iniziale = (1/2)Mv^2 + (1/2)mv^2

p_finale = M(1+m/M)v per ipotesi, la particella di massa M ha velocita'
v(1+m/M), la particella di massa m ha velocita' = 0
E_finale = (1/2)M (1+m/M)^2 v^2

E_finale = (1/2)Mv^2 + (1/2)mv^2(2+ m/M)

sempre maggiore di quella iniziale.


> in particolare
> troviamo a pg. 20:
>
> As already verified by other authors, the experimental data for the U-
> chain shows a good convergence with the energy conserving approach,
> independently of the type of chain. There exists continuous interference
> (no broken contact) during the motion and, consequently, the dissipative
> effects of explicit collisions, even in a discrete chain, are negligible.
> In addition,one-dimensional model is a good approach for this
> configuration.
>
> L'energia si conserva, i risultati sono in accordo coi dati sperimentali.
> Le perdite dissipative per urti fra elementi della catena sono
> trascurabili.

Se i dati sperimentali dicono che l'energia si conserva ok, ma
l'aromentazione "There exists continuous interference (no broken
contact) during the motion and, consequently, the dissipative effects of
explicit collisions, even in a discrete chain, are negligible." non mi
convince.

Da una parte non mi pare funzioni nel caso di una catena dapprima
attorcigliata su un tavolo di cui si lascia cadere inizialmente un capo
(e questo mi sembra il ragionamento di sopra, fatto al contrario: se
prendo una massa ferma e la aggiungo improvvisamente ad una massa in
movimento, imponendo che la q.d.m si conservi, l'energia deve diminuire).

Poi non vedo il passaggio al limite nel caso di elementi della catena
sempre piu' piccoli (che deve darci il "(no broken contact) during the
motion"). Nel thread "Sbarra che cade e si blocca ad un estremo" viene
che la frazione persa di energia non dipende dalla lunghezza della
sbarra, vedi messaggio di Elio
https://groups.google.com/d/msg/it.scienza.fisica/wBONz6PoW7o/J6cu9qgUAQAJ
(ho preso il risultato per buono senza fare il calcolo).

>
> Ci sarebbe altro da aggiungere, ma non qua, il thread sta diventando
> difficilmente gestibile.
>


Puoi aprirne un altro ;-)

MM

unread,
Aug 26, 2019, 3:30:02 PM8/26/19
to
On Sat, 24 Aug 2019 03:04:38 -0700, Wakinian Tanka wrote:


>> Un vincolo anolonomo non conserva necessariamente energia,
>
> L'energia si conserva *sempre*.
Ancora una volta, falso.
https://www.math.unipd.it/~fasso/research/papers/ConsMovEn.pdf

Energy is in general not conserved for mechanical nonholonomic systems
with affine constraints.

> Ma il fatto che si conservi "l'energia meccanica" non implica che questa
> non sia in parte immagazzinata come, ad es, energia potenziale elastica
> nel metallo. Avrebbe dovuto come minimo precisarlo.

Hai fatto il conto col risultato dell'integrale che ti ho fornito?

JTS

unread,
Aug 26, 2019, 3:35:03 PM8/26/19
to
Am 26.08.2019 um 20:55 schrieb MM:

>
> L'energia si conserva, i risultati sono in accordo coi dati sperimentali.
> Le perdite dissipative per urti fra elementi della catena sono
> trascurabili.

I nostri messaggi si sono incrociati, ti ho risposto piu' sotto nel
thread (seconda parte del post delle 19:16).

>
> Se non sei convinto, puoi scrivere un articolo che dimostri il contrario
> e poi farlo pubblicare, meglio ancora se nella stessa rivista.
>

Come ho scritto in quel post: se l'esperimento mostra che l'energia si
conserva, vuol dire che si conserva. Non vuol dire pero' che i
ragionamenti fatti per giustificare l'osservazione sperimentale siano
validi.

Giorgio Pastore

unread,
Aug 26, 2019, 5:10:02 PM8/26/19
to
Il 26/08/19 19:27, JTS ha scritto:
Certo, c'e' un fattore 2 di troppo. Ma è interessante che la dipendenza
dalle variabili del problema sembra esserci tutta. Quando avrò tempo
(quando?) potrei tornaci sopra per capire da dove viene il fattore 2.

Intanto, giusto per dare un paio di ulteriori input, trovo interessante
questa schematizzazione sperimentale del problema:

https://www.youtube.com/watch?v=i9gLi4pBgpk

Inoltre, un'analisi della situazione con un interessante riassunto della
storia del problema è in
https://www.semanticscholar.org/paper/Newtonian-Analysis-of-a-Folded-Chain-Drop-Mungan/01e860fa3510ab48fe9a0a109278dad692e36612

Insomma, è un problema "classico". Ma di quelli tosti.

Wakinian Tanka

unread,
Aug 26, 2019, 5:55:02 PM8/26/19
to
Il giorno lunedì 26 agosto 2019 21:30:02 UTC+2, MM ha scritto:
> On Sat, 24 Aug 2019 03:04:38 -0700, Wakinian Tanka wrote:
>
> > L'energia si conserva *sempre*.
>
> Ancora una volta, falso.
> https://www.math.unipd.it/~fasso/research/papers/ConsMovEn.pdf
> Energy is in general not conserved for mechanical nonholonomic systems
> with affine constraints.
>
Scusa, come fai a non aver afferrato che sto parlando dell'energia totale e non di quella /meccanica/ soltanto?


Era solo questo che intendevo puntualizzare: che nell'articolo di De Sousa ed in questo thread (all'inizio io pure ho commesso questo errore) quando si parla di "energia" si dovrebbe specificare meglio di *quale* energia si sta parlando; io sono solo un pischello senza nemmeno una laurea, gli autori dell'articolo citato non lo sono.

--
Wakinian Tanka
0 new messages